Download as pdf or txt
Download as pdf or txt
You are on page 1of 110

Copyright © 2014 Delhi Academy of Medical Sciences, All Rights Reserved.

1/110
Test Information
Test Name PGI Mock Mid Year Total Questions 250

Test Type Examination Difficulty Level Difficult

Total Marks 250 Duration 180minutes

Test Question Language:- ENGLISH


(1). Neural ectoderm derivatives include?

a. Iris stroma

b. Ciliary epithelium

c. Corneal stroma

d. Corneal epithelium

e. Sphincter and dilator pupillae muscles

Solution. BE
Explanation: The major development of the eye takes place between week 3 and week 10 and involves ectoderm, neural crest cells, and
mesenchyme. The neural tube ectoderm gives rise to the retina, the iris and ciliary body epithelia, the optic nerve, the smooth muscles of
the iris, and some of the vitreous humour. Surface ectoderm gives rise to the lens, the conjunctival and corneal epithelia, the eyelids, and
the lacrimal apparatus. The remaining ocular structures form from mesenchyme.

Correct Answer. b,e

(2). Which among the following are the branches of internal iliac artery?

a. Inferior vesical artery

b. Inferior epigastric artery

c. Iliolumbar artery

d. Internal pudendal artery

e. Obturator artery

Solution. ACDE
Explanation: Inferior epigastric artery is a branch of the external ilac artery

Correct Answer. a,c,d,e

(3). Breast is supplied by?

a. Lateral thoracic artery

b. Internal mammary artery

c. Musculophrenic artery

d. Superior thoracic artery

e. Superior epigastric artery

Solution. ABD
Explanation: The arterial supply to the breast is primarily derived from branches of the internal thoracic (mammary) artery, intercostal
arteries, and the lateral thoracic artery. Superficially, arterial branches of the internal and lateral thoracic arteries arborize across the
breast and send perforating branches deep into the breast parenchyma. Along the posterior (deep) margin of the breast, branches of
theintercostal arteries course along the pectoralis and serratus anterior muscles and send perforating branches through chest wall
musculature and out into the deep breast parenchyma. The internal thoracic artery (internal mammary artery) is the dominant artery
supplying the breast, and its branches supply the medial and central breast parenchyma. The lateral thoracic artery supplies the
superolateral breast parenchyma. Branches of the subclavian and axillary arteries, including the thoraco-acromial artery, the subscapular
artery, and the thoracodorsal artery, often supply to a portion of thesuperior breast parenchyma. Branches of the musculophrenic artery,
a continuation of the internal thoracic artery, supply to a variable portion of the inferior breast. The anterior and posterior intercostal
arteries have branches that perforate through chest wall muscles to supply the deep central breast parenchymal tissues.

Copyright © 2014 Delhi Academy of Medical Sciences, All Rights Reserved. 2/110
Correct Answer. a,b,d

(4). True about epiglottis is/are?

a. Contains serous glands

b. Blood supply derived from superior thyroid artery

c. Lymphatics drain to deep cervical lymph nodes

d. Elastic cartilage forms epiglottis

e. Sensory supply derived from vagal nerve branches

Solution. BCDE
Explanation: Epiglottis
• Develops from the 4th pharyngeal arch
• The epiglottis is a cartilaginous flap that covers the laryngeal inlet upon swallowing,
composed of elastic cartilage
• The epiglottis has a lingual and laryngeal surface, the former is consistent with the
oropharynx posteriorly, and is covered in non-keratinized stratified squamous epithelium
• The laryngeal surface consists of respiratory epithelium, which consists of pseudostratified
ciliated epithelium with goblet cells (which secrete mucus), and is therefore consistent with
the rest of the laryngeal epithelium. This surface forms the anterior wall of the laryngeal
vestibule. There is a pre-epiglottic space between the lingual surface of epiglottis and
thyrohyoid membrane, this space contains the adipose tissue.
• The epiglottis is one of the three large unpairedlaryngeal cartilages, the other two being the
book shaped thyroid and the signet ring shaped cricoid cartilages
• The epiglottis gains its blood supply from the superior laryngeal artery, which is a branch of
the superior thyroid artery (a branch of the external carotid artery).
• Lymphatic drainage of the epiglottis occurs along the superior laryngeal artery that drains to
the deep cervical lymph nodes. Venous drainage is from the superior and inferior laryngeal
veins that follow the same course as the arteries, and drain into the internal jugular vein and
the left brachiocephalic vein respectively.
• The recurrent laryngeal nerves enter the larynx by passing in the groove between the
trachea and esophagus. The nerves innervate the muscles of the larynx (aside from the
cricothyroid). The sensory supply of the epiglottis comes from the internal laryngeal branch
of superior laryngeal nerve (branch of vagus nerve).

Correct Answer. b,c,d,e

(5). Not correct match regarding the lymphatic supply of tongue is/are?

a. Tip—submandibular LNs

b. Tip—Submental LNs

c. Anterior 2/3rd—submandibular LN

d. Posterior 1/3rd--- submandibular LNs

e. Posterior 1/3rd—Jugulo-omohyoid LNs

Solution. BD
Explanation: The tip of tongue drains to the submental nodes. The left and right halves of the anterior two-thirds of the tongue drains to
submandibular lymph nodes, while the posterior onethirds of the tongue drains to the jugulo-omohyoid nodes.

Correct Answer. b,d

(6). Anterior interosseous nerve supplies which among the following?

a. Flexor pollicis longus

b. Flexor digitorum superficialis

c. Medial part of flexor digitorum profundus

d. Lateral part of flexor digitorum profundus

e. Pronator quadratus

Copyright © 2014 Delhi Academy of Medical Sciences, All Rights Reserved. 3/110
Solution. ADE
Explanation: Anterior Interosseous Nerve (AIN)
• Origin: From median nerve between two heads of pronator teres
• Descends on the interosseous membrane
• Supplies: Muscular branches to flexor pollicis longus, Lateral ½ of flexor digitorum profundus and pronator quadratus
• Also gives articular branches to wrist, inferior radio-ulnar joint and intercarpal joints

Correct Answer. a,d,e

(7). True statement(s) about parotid gland is/are?

a. 2nd largest salivary gland

b. Stenson’s duct opens behind upper 3rd molar

c. Divided by branches of facial nerve

d. Lymph nodes are embedded in the gland

e. Only benign tumours arise from parotid gland

Solution. CD
Explanation: Parotid gland
• Largest salivary gland
• Purely serous gland
• The facial nerve forms the parotid plexus within the gland and separates the glandular tissue
partially into superficial and deep layers ("lobes").
• Duct opens opposite upper 2nd molar
• Lymph nodes are embedded in the gland
• Benign as well as malignant tumours may arise

Correct Answer. c,d

(8). Following are tributaries of Inferior vena cava

a. Left and right common iliac veins

b. Left gonadal vein

c. Left renal vein

d. Right suprarenal vein

e. Hepatic veins

Solution. ACDE
Explanation: Inferior vena cava is formed by the union of left and right common iliac veins opposite L5 vertebra. Its tributaries are
- Left and right common Iliac veins
- 3rd and 4th pair of lumbar veins (1st and 2nd unite to form the Ascending lumbar vein)
- Right gonadal vein
- Right and left renal veins
- Right suprarenal vein
- Right and left phrenic veins
- Hepatic veins (2 sets)- upper right, left and middle; lower 2 veins on left and 1 on right side

Correct Answer. a,c,d

(9). On damage to medial part of superior orbital fissure, which among the following will NOT be damaged?

a. Superior ophthalmic vein

b. Frontal nerve

c. Lacrimal nerve

d. Nasociliary nerve

e. Inferior ophthalmic vein

Copyright © 2014 Delhi Academy of Medical Sciences, All Rights Reserved. 4/110
Solution. ABCD

Correct Answer. a,b,c,d

(10). Non-keratinized stratified squamous epithelium may be found in?

a. Esophagus

b. Rectum

c. Anal canal

d. Vagina

e. Prepuce

Solution. ACD
Explanation: Examples of non-keratinized stratified squamous epithelium include corneal epithelium, lining mucosa of oral cavity,
esophagus, anal canal, ectocervix, vagina, foreskin, and the internal portion of the lip

Correct Answer. a,c,d

(11). Not the indication(s) for one-lung ventilation is/are?

a. Lung cyst

b. Lung abscess

c. Lung transplant

d. Liver transplant

e. Broncho-pleural fistula

Copyright © 2014 Delhi Academy of Medical Sciences, All Rights Reserved. 5/110
Solution. D
Explanation: One-lung ventilation (OLV) is employed toenable unilateral ventilation if the contralateral airways are to be opened, to
prevent secretions, pus or blood spilling from one side to the other and to facilitate surgery. Indications include
• Bronchopleural fistula
• Pulmonary suppurative disease – abscess
• Bronchiectasis
• Endobronchial haemorrhage
• Lung transplantation
• Bronchial transection/tear
• Bronchoplastic surgery (e.g. sleeve resection)
• Bullous lung disease
• Lung cyst
• Undrained pneumothorax
• Pulmonary resection (lobectomy, pneumonectomy)
• Thoracoscopic surgery
• Oesophageal surgery
• Pleurectomy
• Surgery for thoracic aortic disease
• Spinal surgery

Correct Answer. d

(12). Complications of PEEP include:

a. Increased intracranial pressure

b. Decreased cardiac output

c. Increased renal output

d. Pneumothorax in an emphysematous patient

e. Hypercarbia

Solution. ABD
Explanation: Complications of PEEP include
• Decreased venous return
• Decreased cardiac output
• Pneumothorax (esp in emphysema and overexpanded lungs)
• Increased intracranial pressure
• Barotrauma
• Co2 retention
• Increased dead space

Correct Answer. a,b,d

(13). Component(s) of advanced cardiovascular life support (ACLS) according to AHA 2015 guidelines is/are?

a. Rate of chest compression 100-120/min

b. One breath every 10 seconds

c. Vasporessin preferred over epinephrine

d. Depth of chest compression 2 inches/5 cm

e. Capnography no longer followed to confirm ET tube placement

Copyright © 2014 Delhi Academy of Medical Sciences, All Rights Reserved. 6/110
Solution. AD

Copyright © 2014 Delhi Academy of Medical Sciences, All Rights Reserved. 7/110
Correct Answer. a,d

(14). Correct statement(s) regarding remifentanil include:

a. Metabolised by plasma acetylcholinesterase

b. Has a short T1/2

c. More potent than Alfentanyl

d. Dose reduction needed in hepatic failure

e. Duration of action is prolonged as compared to Alfentanyl

Solution. BC
Explanation: Remifentanil
• Useful in day care surgery
• A selective mu-opiod-agonist with good analgesic activity
• Short elimination half-life of 6-11 min (shorter than Alfentanyl)
• 15-20 times potent than Alfentanyl
• Not metabolised by plasma cholinesterases. Esterase activity present in RBCs
• Dose is unchanged in renal and hepatic failure

Correct Answer. b,c

(15). Not a useful indication of Ketamine is/are?

a. Refractory cyanotic spells

b. Induction anesthetic in Head injury

c. Induction anesthetic in Squint surgery

d. Induction anesthetic in hypovolemia

e. Induction anesthetic in Pheochromocytoma

Solution. BCE
Explanation: Ketamine is a short-acting, potent analgesic and anesthetic given by IV route. Since it increases sympathetic tone, produces
arterial constriction and increases intra-arterial pressure, hence it is unsafe in pheochromocytoma surgery, It also increases intracranial
and intraocular pressures hence not preferred in head injury and glaucoma/squint surgery. But it is safe and preferred induction agent in
hypovolemia. It also aborts refractory cyanotic spell in TOF patients by decreasing R→L shunt and also produces bronchodilatation in
severe status asthmaticus.

Correct Answer. b,c,e

(16). All are true regarding Urea cycle, except:

a. Urea is formed from ammonia

b. Rate limiting enzyme is Ornithine Transcarbamoylase

c. Requires energy expenditure

d. Malate is by-product of urea cycle

e. One nitrogen of urea comes from aspartate

Solution. BD
Explanation: Rate limiting enzyme of urea cycle is Carbamoyl phosphate synthetase-I By-product of urea cycle is Fumarate (Not Malate).
Fumarate further forms malate in TCA cycle.
Other options are correct.

Correct Answer. b,d

(17). Not seen in mitochondrial DNA?

a. 10% of cellular DNA

b. Codes for respiratory chain proteins

c. Identical to nuclear DNA

Copyright © 2014 Delhi Academy of Medical Sciences, All Rights Reserved. 8/110
d. Full of untranslated sequences

e. High mutation rate

Solution. BE
Explanation:

Correct Answer. b,e

(18). Complication(s) of gene therapy may include?

a. Genotoxicity

b. Cytotoxicity

c. Phenotoxicity

d. Gene silencing

e. Immunotoxicity

Solution. ACDE

Correct Answer. a,c,d,e

(19). Biomarker(s) used for monitoring response to therapy in Pompe disease include(s)?

a. Blood glucose

b. Urinary glucose

c. Urinary glucose tetrasaccharides

Copyright © 2014 Delhi Academy of Medical Sciences, All Rights Reserved. 9/110
d. Urinary acetoned

e. Ejection fraction on echocardiography

Solution. C
Explanation: Urinary glucose tetrasaccharides are elevated in the urine of affected patients, and levels are extremely high in infantile
patients. This biomarker is valuable for diagnosis and monitoring response to therapy in Pompe disease.

Correct Answer. c

(20). Coarse facial features with corneal clouding is seen in which type(s) of mucopolysaccharidoses?

a. Type I-H

b. Type I-S

c. Type II

d. Type III

e. Type VI

Solution. ABE
Explanation:

Correct Answer. a,b,e

(21). Enzymes having proof-reading function in PCR?

a. Taq polymerase

b. Pfu polymerase

c. Pwo polymerase

d. Tth polymerase

e. Bestaq DNA polymerase

Solution. BCE
Explanation: Taq polymerase does not have proof-reading activity. Alternative polymerases with
proof-reading activity:
• Pfu polymerase: from Pyrococcusfuriosus
• Pwo polymerase: from Pyrococcuswoesei
• Bestaq DNA polymerase
• Kodaq DNA polymerase

Correct Answer. b,c,e

Copyright © 2014 Delhi Academy of Medical Sciences, All Rights Reserved. 10/110
(22). Which among the following statements are TRUE regarding Locked Nucleic Acid (LNA)?

a. Also known as Inaccessible RNA

b. Are responsible for decreasing the sensitivity of DNA and FISH probes

c. Miravirsen is a drug based on LNA

d. Occur naturally in specific parts of X chromosome

e. Facilitate normalization of Tm in various biotechnology procedures

Solution. ACE
Explanation: Locked nucleic acid (LNA):
- Also known as Inaccessible RNA
- A type of synthetic RNA nucleotide, where the ribose moiety is modified by an extra bridge
connecting the 2’oxygen & 4’carbon.
- Useful to increase sensitivity & specificity of DNA microarrays, FISH probes & PCR assays.
- Facilitates Tm normalization, increases the thermal stability of duplexes
- Miravirsen, a new anti-HCV drug targeting miR-122 is being developed based on this technology.

Correct Answer. a,c,e

(23). Hyperphenylalaninemia is seen in?

a. Phenylalanine hydroxylase deficiency

b. Phenylalanine hydroxylase overactivity

c. Dihydrobiopterine reductase deficiency

d. Homogentisic oxidase deficiency

e. Dihydrobiopterin reductase excess

Solution. AC

Correct Answer. a,c

(24). Diffusion of a substance across biological membrane depends upon?

a. Gradient

b. Temperature

c. Charge

Copyright © 2014 Delhi Academy of Medical Sciences, All Rights Reserved. 11/110
d. Molecular weight

e. Chemical composition

Solution. ABC
Explanation: The following affect net diffusion of a substance:
(1) concentration gradient across the membrane
(2) electrical potential across the membrane: solutes move toward the solution that has the
opposite charge. The inside of the cell usually has a net negative charge;
(3) permeability coefficient of the substance for the membrane;
(4) hydrostatic pressure gradient across the membrane: increased pressure will increase the rate and
force of the collision between the molecules and the membrane; and
(5) temperature, since increased temperature will increase particle motion and thus inc
frequency of collisions between external particles and the membrane

Correct Answer. a,b,c

(25). Glucose epimers include(s)?

a. Fructose

b. Lactose

c. Mannose

d. Galactose

e. Trehalose

Solution. CD
Explanation: Isomers differing as a result of variations in configuration of the carbon atoms 2, 3, and 4 of glucose are known as epimers.
Biologically, the most important epimers of glucose are mannose (epimerized at carbon 2) and galactose (epimerized at carbon 4)

Correct Answer. c,d

(26). True regarding sphingomyelins is/are?

a. Found in outer leaflet

b. Found in inner leaflet

c. Present in lipid rafts of the plasma membrane

d. Play a role in cell signalling

e. Play a role in apoptosis

Solution. ACDE
Explanation: Sphingomyelins are found in the outer leaflet of the cell membrane lipid bilayer and are particularly abundant in specialized
areas of the plasma membrane known as lipid rafts. They are also found in large quantities in the myelin sheath that surrounds nerve
fibers. They are believed to play a role in cell signalling and in apoptosis. Sphingomyelins contain no glycerol, and on hydrolysis they yield
a fatty acid, phosphoric acid, choline, and sphingosine.

Correct Answer. a,c,d,e

(27). Major lipoproteins present in Apo E include?

a. VLDL

Copyright © 2014 Delhi Academy of Medical Sciences, All Rights Reserved. 12/110
b. LDL

c. IDL

d. HDL

e. Lp(a)

Solution. ABD

Correct Answer. a,b,d

(28). Copper containing enzymes is/are:

a. Superoxide dismutase

b. Cytochrome oxidase

c. Myeloperoxidase

d. Tyrosinase

e. Amine oxidase

Solution. ABDE
Explanation: Copper containing enzymes include: Superoxide dismutase, Cytochrome oxidase, Tyrosinase, Amine oxidase.
Myeloperoxidase (MPO) is an iron containing enzyme, not copper containing.

Correct Answer. a,b,d,e

(29). Which of the following diseases are correctly matched with their deficient enzymes

a. Homocystinuria- Cystathione beta synthase

b. Methylmalonic acidemia- Methylmalonyl CoA mutase

c. Maple syrup urine disease- Branched chain alpha keto acid dehydrogenase

d. Oculocutaneous albinism- Tyrosinase

e. Phenylketonuria- Phenylalanine hydroxylase

Solution. ABCDE
Explanation: All are correct. Deficiency of dihydropteridine reductase or any of the enzymes needed for BH4 synthesis can also result in
hyperalaninemia.

Correct Answer. a,b,c,d,e

Copyright © 2014 Delhi Academy of Medical Sciences, All Rights Reserved. 13/110
(30). Vitamin C dependent mixed function oxidase activity is needed for which among the following amino acids?

a. Proline

b. Tyrosine

c. Lysine

d. Alanine

e. Tryptophan

Solution. AC
Explanation: Peptidyl hydroxyproline and hydroxylysine are formed by hydroxylation of peptidyl
proline or lysine in reactions catalyzed by mixed-function oxidases that require vitamin C as cofactor.

Correct Answer. a,c

(31). A 20 yr old anemic man is found to have an abnormal form of Beta globin that is 172 amino acids long rather than the 141 found in the
normal protein. Which of the following point mutations is consistent with this abnormality?

a. UAA → CAA

b. UAA → UAG

c. CGA → UGA

d. GAU → GAC

e. GCA→ GAA

Solution. A
Explanation: Mutating the normal stop codon for beta globin from UAA to CAA causes the ribosome
to insert a glutamine at that point. It will continue extending the protein chain until it comes upon the next stop codon further down the
message, resulting in an abnormally long protein.
A change from UAA to UAG would simply change one termination codon to another and would have no effect on the protein. The
replacement of CGA (arginine) with UGA (stop) would cause the protein to be too short.
GAU and GAC both encode aspartate and would cause no change in the protein. Changing GCA (alanine) to GAA (glutamate) would not
change the size of the protein product.

Correct Answer. a

(32). Part of eukaryotic DNA contributing to polypeptide synthesis?

a. Exon

b. Enhancer

c. Leader sequence

d. tRNA

e. ncRNA

Solution. AB
Explanation: Exons are the coding regions of a gene sequence
• Enhancers are special cis-acting DNA sequences that increase the rate of initiation of
transcription of eukaryotic genes by RNA POL II
• Leader sequence is the sequence at 5 end of an mRNA that is not translated into a protein.
• ncRNA is Non-Coding RNA. They code for tRNA, rRNA, microRNA, snoRNA, siRNA, piRNA and the long ncRNA.

Correct Answer. a,b

(33). Mutations that completely disrupt the function of the gene are used in which of the following techniques?

a. Knock out

b. Non sense mutation

c. RFLP

d. Targeted gene disruption

Copyright © 2014 Delhi Academy of Medical Sciences, All Rights Reserved. 14/110
e. Knock in

Solution. AD
Explanation: Both Targeted gene disruption & Knock-out are examples of mutations which disrupt the function of the gene.

Correct Answer. a,d

(34). Treatment modalities effective in rosacea may include?

a. Doxycycline

b. Dermabrasion

c. Apremilast

d. Probiotics

e. Metronidazole

Solution. ABE
Explanation: Management of rosacea:
• Topical: Metronidazole cream (topical DOC), Erythromycin, clindamycin.
• Systemic: Doxycycline (oral DOC), Metronidazole, Tetracycline, and Minocycline, isotretinoin (topical and systemic therapy can be
combined)
• Rhinophyma: Surgical approach (dermabrasion, laser).

Correct Answer. a,b,e

(35). Agent(s) associated with allergic contact dermatitis?

a. Poison ivy

b. Nickel

c. Gold

d. Paraphenylene diamine

e. Rubber

Solution. ABDE
Explanation: Allergic contact dermatitis is a type IV hypersensitivity reaction. Common causes
include:
• Nickel (earrings, jeans buttons, coins) → MC allergen.
• Parthenium hysterophorus (congress grass) is commonest cause of ABCD in India
• Latex (health professionals)
• Fragrance mixes (cosmetics): “Berloque dermatitis” is pendant-like linear streaks of
pigmentation on the neck, face, arms, or trunk.
• Poison ivy
• Rubber
• Paraphenylene diamine (hair dye)
• Chromate (in cement)

Correct Answer. a,b,d,e

(36). Not seen in atopic dermatitis?

a. Pruritus

b. Extensor surfaces involved in adolescents

c. Relapses are common

d. Dennie-Morgan folds

e. Nipple sparing

Copyright © 2014 Delhi Academy of Medical Sciences, All Rights Reserved. 15/110
Solution. BE
Explanation: Clinical features of atopic dermatitis/eczema

Correct Answer. b,e

(37). Risk factor(s) for Norwegian scabies may include?

a. Leprosy

b. HIV

c. Psoriasis

d. Down syndrome

e. Marasmus

Solution. ABD
Explanation: The Norwegian variant of human scabies is highly contagious andoccurs mainly in individuals who are cognitively and
physically debilitated,particularly those who are institutionalized and those with Downsyndrome; in patients with poor cutaneous
sensation (leprosy, spinabifida); in patients who have severe systemic illness (leukemia, diabetes);and in immunosuppressed patients (HIV
infection). Affected individualsare infested by myriad mites that inhabit the crusts andexfoliating scales of the skin and scalp.

Correct Answer. a,b,d

(38). Antibodies seen in bullous pemphigoid are directed against?

a. BP180

b. BP230

c. Alpha-actinin 4

d. Alpha-6-integrin

e. Desmoglein-4

Copyright © 2014 Delhi Academy of Medical Sciences, All Rights Reserved. 16/110
Solution. ABD
Explanation: Bullous pemphigoid is primarily an autoimmune condition, with the hallmark finding of circulating and tissue antibodies
directed against specific structural elements near the basement membrane. The autoantibodies are specifically directed against two
hemidesmosomal antigens, BP180 and BP230. While BP180 (also called BPAG-2) is a transmembrane adhesion protein, BP230 (also called
BPAG-1) is a cytoplasmic protein involved in the anchorage of intermediate filaments to the cytoskeleton. Majority of patients with BP
manifest IgG type autoantibodies directed against both BP180 and BP230, although the anti-BP180 antibodies play a more important role
in producing cutaneous blisters. Rarely, autoantibodies may occur against other antigens in the basement membrane zone, like laminin-5
and alpha-6-integrin

Correct Answer. a,b,d

(39). True regarding basal cell nevus syndrome is/are?

a. Also called Gorlin-Goltz syndrome

b. Autosomal recessive

c. Multiple basal cell carcinomas seen

d. Onset in middle age

e. Mental retardation seen

Solution. AC
Explanation: Basal cell nevus syndrome (BCNS), also called Gorlin-Goltz syndrome is an autosomal dominant, multisystem disorder. Most
patients have mutations in the tumour suppressor gene, PTCH-1 present on chromosome 9q, with de novo mutations reported in 60%
cases. Multiple basal cell carcinomas are hallmark, with onset before 20 years age in most cases. Other common findings include
odontogenic keratocysts, palmo-plantar pits, calcification of falx cerebri, increased risk of medulloblastomas, rib anomalies, macrocephaly
and abnormal facies.

Correct Answer. a,c

(40). Not true regarding sudden sensorineural hearing loss?

a. Occurs within 72 hours

b. Bilateral loss

c. Never accompanied with vertigo

d. May have vascular etiology

e. Can be psychogenic

Solution. BC
Explanation: Sudden SNHL is defined as 30 dB or more of SNHL over atleast three contiguous frequencies occurring within a periodof 3
days or less. Mostly it is unilateral. It may be accompaniedby tinnitus or temporary spell of vertigo.
Causes: Remember the mnemonic “InThe Very Ear Too No Major Pathology.”
1. Infections. Mumps, herpes zoster, meningitis, encephalitis,syphilis, otitis media.
2. Trauma. Head injury, ear operations, noise trauma, barotrauma,spontaneous rupture of cochlear membranes.
3. Vascular. Haemorrhage (leukaemia), embolism or thrombosisof labyrinthine or cochlear artery or their vasospasm.They may be
associated with diabetes, hypertension,
polycythaemia,macroglobinaemia or sickle cell trait.
4. Ear (otologic). Ménière’s disease, Cogan’s syndrome, large vestibular aqueduct.
5. Toxic. Ototoxic drugs, insecticides.
6. Neoplastic. Acoustic neuroma. Metastases in cerebellopontineangle, carcinomatous neuropathy.
7. Miscellaneous. Multiple sclerosis, hypothyroidism,sarcoidosis.
8. Psychogenic

Correct Answer. b,c

(41). Identify the correct statement(s) regarding functional aphonia:

a. Seen in adolescent males

b. Communicate with whispers

c. Vocal cords are adducted

d. Abduction of vocal cords seen on coughing

e. Psychotherapy is effective

Copyright © 2014 Delhi Academy of Medical Sciences, All Rights Reserved. 17/110
Solution. BE
Explanation: Functional aphonia
It is a functional disorder mostly seen in emotionally labilefemales in the age group of 15–30 years. Aphonia is usuallysudden and
unaccompanied by other laryngeal symptoms.Patient communicates with whisper. On examination, vocalcords are seen in abducted
position and fail to adduct onphonation; however, adduction of vocal cords can be seenon coughing, indicating normal adductor function.
Eventhough patient is aphonic, sound of cough is good. Treatmentgiven is to
reassure the patient of normal laryngealfunction and psychotherapy.

Correct Answer. b,e

(42). Structures preserved in modified radical neck dissection:

a. SCM muscle

b. Submandibular gland

c. Omohyoid muscle

d. Internal jugular vein

e. Accessory nerve

Solution. ADE
Explanation: Three structures are preserved in modified radical neck dissection:
• Accessory nerve
• SCM muscle
• Internal jugular vein

Correct Answer. a,d,e

(43). True statement(s) regarding epistaxis is/are?

a. In 90% cases, bleeding arises from Kiesselbach’s plexus

b. Trotter’s method is best for initial management

c. Cautery should be done under GA

d. Injury to nasal mucosa is less with anterior nasal packing than balloon compression

e. Antibiotics are given if anterior pack left in nose for more than 48 hr

Solution. ABCE
Explanation: Epistaxis
• In 90% cases, bleeding arises from Kiesselbach’s plexus (Little’s area): Situated in anteroinferior
part of nasal septum, just above the vestibule
• As soon as bleeding begins, patient is advised to pinch the nose with thumb and index finger
for 5 min to compress the vessels
• Patient is made to sit in sitting position, leaning forwards with mouth open (Trotter’s
method). Cold compresses may also be applied to the nose to produce vasoconstriction of
the vessels
• Anterior nasal pack is applied if bleeding persists. If pack requires to be placed for greater
than 24 hr, prophylactic antibiotics are indicated
• A balloon tamponade can be used as an alternative to anterior nasal pack. This method is
less traumatic and preferred in bleeding/coagulation disorders
• In refractory cases, Bipolar cautery under GA may be required.

Correct Answer. a,b,c,e

(44). In bilateral abductor paralysis, which of the following is seen?

a. Vocal cord in paramedian position

b. Voice is affected early

c. Stridor & dyspnea occur

d. Vocal cord lateralization is done

e. Hoarseness occur

Copyright © 2014 Delhi Academy of Medical Sciences, All Rights Reserved. 18/110
Solution. ACD
Explanation: Bilateral Abductor Paralysis:
• Etiology: Neuritis, trauma, Post-thyroidectomy
• Cords lie in median or paramedian position
• Dyspnea & stridor seen but voice is good
• Rx: Acute emergency needs tracheostomy
• For correction, Lateralisation procedures are done eg, Arytenoidectomy(Woodman’s op),
Type 2 thyroplasty, Cordectomy & nerve- muscle implant

Correct Answer. a,c,d

(45). A patient presents with otitis media with effusion. Identify the true statement?

a. Most common cause of conductive hearing loss in children

b. Type C tympanogram in early stages

c. Flat tympanogram is seen

d. Eustachian tube dysfunction is common

e. Less common in cleft palate

Solution. ABCD
Explanation: Otitis media with effusion (OME)
• Synonyms: Serous otitis media, Glue ear, Secretory otitis media, Middle ear effusion
• Considered MC cause of conductive hearing loss in children (Nelson 20th Ed)
• An insidious condition characterised by accumulation of non purulent effusion in middle ear
cleft (Mostly thick and viscid but can be thin and serous as well)
• The fluid is sterile.
• Seen in young school going children (5-8 year)
• Eustachian tube dysfunction is major factor in pathogenesis; with some role of allergy and
viral infections as well. Risk is increased in cleft palate, adenoid and tonsillar hypertrophy.
• Hearing loss is mild to moderate and doesn’t exceed 40 dB
• Otoscopy: Dull, opaque TM with loss of light reflex. Retractions of TM with some restriction
of movement may sometimes be seen.
• Tympanogram: Early stages show Type C curve, whereas established OME has flat or dome
shaped curve (i.e. Type B curve)
• Medical therapy: Topical decongestants, antihistamines, frequent valsalva and antibiotics
• Surgical therapy:
- Myringotomy and aspiration of fluid (Commonly done)
- Grommet insertion
- Tympanotomy and cortical mastoidectomy

Correct Answer. a,b,c,d

(46). Symptom(s) of deviated nasal septum may include?

a. Unilateral nasal obstruction

b. Bilateral nasal obstruction

c. Hyperosmia

d. Saddle nose

e. Recurrent sinusitis

Solution. ABDE
Explanation: Deviated nasal septum (DNS)symptoms may include
1. Nasal obstruction. Depending on the type of septal deformity,obstruction may be unilateral or
bilateral.
2. Headache. Deviated septum, especially a spur, may presson the lateral wall of nose giving rise to
pressure headache.
3. Sinusitis. Deviated septum may obstruct sinus ostia resultingin poor ventilation of the sinuses.
Therefore, it forms animportant cause to predispose or perpetuate sinus infections.
4. Epistaxis. Mucosa over the deviated part of septum isexposed to the drying effects of air currents
leading toformation of crusts, which when removed causes bleeding.
5. Anosmia. Failure of the inspired air to reach the olfactoryregion may result in total or partial loss
of sense of smell.
6. External deformity. Septal deformities may be associatedwith deviation of the cartilaginous or
both the bony and cartilaginousdorsum of nose, deformities of the nasal tip orcolumella.
7. Middle ear infection. DNS also predisposes to middle earinfection.

Copyright © 2014 Delhi Academy of Medical Sciences, All Rights Reserved. 19/110
Correct Answer. a,b,d,e

(47). Characteristics of conductive hearing loss are all except

a. Negative Rinne’s test

b. Weber lateralized to better ear

c. High frequencies affected more

d. Loss is not more than 60dB

e. Speech discrimination is good

Solution. BC
Explanation: Characteristics of hearing loss

Correct Answer. b,c

(48). True regarding Gradinego syndrome may include?

a. Seen in petrositis

b. 5th CN involved
th
c. 6 CN involved

d. 7th CN involved

e. Intermittent ear discharge

Solution. ABC
Explanation: Gradenigo syndrome is the classical presentation of petrositis and consistsof a triad of (i) external rectus palsy (VIth nerve
palsy),(ii) deep-seated ear or retro-orbital pain (Vth nerve involvement)and (iii) persistent ear discharge.

Correct Answer. a,b,c

(49). Which among the following are the clinical signs seen in Glomus tumour of middle ear?

a. Rising sun appearance

b. Pulsation sign

c. Vestibular decay

d. Brown’s sign

e. None of the above

Solution. ABD
Explanation: 4 signs are seen in glomus tumour of middle ear:
- Pulsatile tinnitus
- Brown’s sign (Blanching of the TM with +ve pressure)
- Pulsation sign
- Rising sun appearance (when tm arises from floor of middle ear)

Correct Answer. a,b,d

Copyright © 2014 Delhi Academy of Medical Sciences, All Rights Reserved. 20/110
(50). Perjury: True statement(s) is/are?

a. Giving wilful false evidence under oath despite knowing it not to be true

b. Trying to bribe the jury

c. Sections 190 to 193 IPC deal with this

d. Liable for punishment

Solution. AD
Explanation: Perjury
• Means giving wilful false or fabricated evidence under oath
• S.191 and 192 of IPC deal with definition and description of perjury
• S.191: Giving false evidence
• S.192: Fabricating false evidence
• S.193: Punishment for perjury/false evidence. Fine or imprisonment extending upto 7 years
or both.

Correct Answer. a,d

(51). True statement(s) regarding plumbismis/are?

a. Facial pallor is the earliest sign

b. Burtonian line seen in 50% patients

c. Encephalopathy may occur

d. Anemia seen

e. Constipation is MC GIT manifestation

Solution. ABCDE
Explanation: All are correct statements.

Correct Answer. a,b,c,d,e

(52). Post-mortem caloricity can be seen in?

a. Septicaemia

b. Tetanus

c. Pontine haemorrhage

d. Sunstroke

e. Rabies

Solution. ABCDE
Explanation: Common causes of PM caloricity
• Pontine haemorrhage
• Sunstroke/heat stroke
• Infections like tetanus, rabies, typhoid, encephalitis, yellow fever
• Bacteremia and septicaemia
• Alcoholism
• Asphyxia

Correct Answer. a,b,c,d,e

(53). Mummification is favoured by:

a. Dry air

b. Humid air

c. Cold environment

d. Drowning

e. Warm air

Copyright © 2014 Delhi Academy of Medical Sciences, All Rights Reserved. 21/110
Solution. AE
Explanation: For mummification to occur, there should be absence of moisture in the air. Instead, there should be continuous action of dry
or warmed air.

Correct Answer. a,e

(54). Which among the following is/are true regarding hypovolemic shock?

a. Cardiac output is decreased

b. Systemic vascular resistance is decreased

c. Systemic vascular resistance is increased

d. PCWP is decreased

e. Venous O2 saturation is decreased

Solution. ACDE

Correct Answer. a,c,d,e

(55). True regarding Takotsubo cardiomyopathy is/are?

a. Stress-induced

b. Seen in adolescents

c. Due to parasympathetic activity

d. Anticoagulation is avoided

e. Recurrence is never seen

Solution. AD
Explanation: TAKOTSUBO CARDIOMYOPATHY
• The apical ballooning syndrome, or stress
women after sudden intense emotional or physical stress.
• The ventricle shows global ventricular dilation with basal contraction, forming the shape of
the narrow-necked jar (takotsubo) used in Japan to trap octopuses.
• Originally described in Japan, it is increasingly recognized elsewhere during emergency
cardiac catheterization and intensive care unit admissions for noncardiac conditions.
• Presentations include pulmonary edema, hypotension, and chest pain with ECG changes
mimicking an acute infarction. The left ventricular dysfunction extends beyond a specific
coronary artery distribution and generally resolves within days to weeks
• May result from intense sympa
innervation, diffuse microvascular spasm, and/or direct catecholamine toxicity. Coronary
angiography may be required to rule out acute coronary occlusion.
• No therapies have been proven beneficia
pulmonary edema, intra
beta blockers rather than selective beta blockade if hemodynamically stable, and
magnesium for arrhythmias related to
• Anticoagulation is generally withheld due to the occasional occurrence of ventricular
rupture.
• While the prognosis is generally good, recurrences have been described in up to 10% of
patients.

Correct Answer. a,d

(56). True regarding eosinophilic granulomatosis with polyangiitis (EGPA) is/are?

Copyright © 2014 Delhi Academy of Medical Sciences, All Rights Reserved. 22/110
a. Same as Churg-strauss syndrome

b. Pulmonary eosinophilia

c. Peripheral blood eosinophilia

d. Mostly seen in adolescents

e. Plasma exchange is the therapy of choice

Solution. ABC
Explanation: Eosinophilic granulomatosis with polyangiitis (EGPA)
• Previously known as allergic angiitis granulomatosis or Churg-Strauss syndrome
• Patients have peripheral as well as pulmonary eosinophilia along with eosinophilic vasculitis,
along with multisystem involvement, including lungs, heart, kidneys and nervous system
• ANCA positive in 2/3rd patients. Corticosteroids are the therapy of choice. Plasma exchange
is not effective.

Correct Answer. a,b,c

(57). True statement(s) regarding multiple sclerosis is/are?

a. More common in men

b. Relapsing-remitting variety is most common

c. During acute attack, nerve conduction continues

d. Visual symptoms present early

e. Urinary and bladder problems present in most patients

Solution. BDE
Explanation: Multiple sclerosis
• MS is approximately threefold more common in women than men. The age of onset is
typically between 20 and 40 years (slightly later in men than in women), but the disease can
present across the lifespan.
• Relapsing/remitting MS (RRMS) accounts for 85% of MS cases at onset and is characterized
by discrete attacks that generally evolve over days to weeks (rarely over hours).
• Bladder dysfunction is present in >90% of MS patients, and in a third of patients, dysfunction
results in weekly or more frequent episodes of incontinence.
• Optic neuritis (ON) presents as diminished visual acuity, dimness, or decreased color
perception (desaturation) in the central field of vision. These symptoms can be mild or may
progress to severe visual loss. Rarely, there is complete loss of light perception. Visual
symptoms are generally early onset and monocular but may be bilateral. Periorbital pain
(aggravated by eye movement) often precedes or accompanies the visual loss.

Correct Answer. b,d,e

(58). Chronic hepatitis B patient with high-infectivity is expected to have which among the following?

a. HBsAg +

b. IgM Anti-HBc+

c. IgG Anti-HBc+

d. HBeAg+

e. Anti-HBs+

Copyright © 2014 Delhi Academy of Medical Sciences, All Rights Reserved. 23/110
Solution. ACD

Correct Answer. a,c,d

(59). True about diagnostic criteria of chronic fatigue syndrome is/are?

a. Fatigue lasts at least for 3 months

b. Fatigue is of new onset

c. It is alleviated by rest

d. Associated with severe obesity

e. Substantial reduction in social and personal activities

Solution. BE
Explanation: Diagnostic criteria for chronic fatigue syndrome
- Fatigue lasts for atleast 6 months
- Fatigue is of new or definite onset
- Fatigue is not the result of organic disease or of continuing exertion
- Fatigue is not alleviated by rest
- Fatigue results in substantial reduction in previous occupational, educational, social and personal activities
- 4 or more of the following symptoms concurrently present for 6 months

Correct Answer. b,e

(60). True statement(s) regarding autoimmune hepatitis is/are?

a. Anti-LKM-1 antibodies seen only in type 1 variety

b. Females more commonly affected

c. Associated with other autoimmune conditions

d. Rarely cause cirrhosis

Copyright © 2014 Delhi Academy of Medical Sciences, All Rights Reserved. 24/110
e. Oral corticosteroids effective in severe cases

Solution. -NA-

Correct Answer. b,c,d,e

(61). Agent(s) effective in the management of acute ischemic stroke is/are?

a. Heparin

b. Warfarin

c. Clopidogrel

d. Aspirin

e. Nitrates

Solution. D
Explanation: Aspirin is the only antiplatelet agent that has been proven to be effective for the acute treatment of ischemic stroke; there
are several antiplatelet agents proven for the secondary prevention of stroke.Numerous clinical trials have failed to demonstrate any
benefit of routine anticoagulation in the primary treatment of atherothrombotic cerebral ischemia, and have also shown an increase in the
risk of brain and systemic hemorrhage. Therefore, the routine use of heparin or other anticoagulants for patients with atherothrombotic
stroke is not warranted.

Correct Answer. d

(62). In hereditary hemorrhagic telangiectasia, true is/are?

a. Hematuria is the most common symptom

b. Hypochromic microcytic anemia is common

c. There is thrombocytopenia

d. The skin lesions are mostly on chest & abdomen

e. Autosomal recessive inheritance

Solution. B
Explanation: Hereditary hemorrhagic telangiectasia is also known as Osler-Rendau-weber disease. It is autosomal dominant in
inheritance. Epistaxis is the most common presentation. The platelet count is usually normal. Microcytic hypochromic anemia seen in
majority of the patients. The skin lesions are mainly seen on the face, hands and feet.

Correct Answer. b

(63). Component(s) of Glasgow-Blatchford score include(s)?

a. Hemoglobin

b. Platelet count

c. BUN

d. Syncope

e. Splenomegaly

Copyright © 2014 Delhi Academy of Medical Sciences, All Rights Reserved. 25/110
Solution. ACD
Explanation: In patients with upper GIT bleeding, discharge from the emergency room with
outpatient management has been suggested for patients with a score known as Glasgow-Blatchford score (possible range 0–23) of 0–1 or
0–2 among patients.

Correct Answer. a,c,d

(64). Which among the following is/are associated with underlying complement pathway defect?

a. Recurrent Neisseria infections

b. Haemolytic uremic syndrome

c. Pyogenic infections

d. Atypical mycobacterial infection

e. Immune thrombocytopenia

Copyright © 2014 Delhi Academy of Medical Sciences, All Rights Reserved. 26/110
Solution. ABC

Correct Answer. a,b,c

(65). A 32-year-old female is seen in the emergency department for acute shortness of breath. A helical CT shows no evidence of pulmonary
embolus, but incidental note is made of dilatation of the ascending aorta to 4.3 cm. All the following are associated with this finding

a. Syphilis

b. Takayasu’s arteritis

c. Giant cell arteritis

d. Rheumatoid arthritis

e. Systemic lupus erythematosus

Solution. ABCD
Explanation: Aortitis and ascending aortic aneurysms are commonly caused by cystic medial necrosis and mesoaortitis that result in
damage to the elastic fibers of the aortic wall with thinning and weakening. Many infectious, inflammatory, and inherited conditions have
been associated with this finding, including syphilis, tuberculosis, mycotic aneurysm, Takayasu’s arteritis, giant cell arteritis, rheumatoid
arthritis, and the spondyloarthropathies (ankylosing spondylitis, psoriatic arthritis, Reiter’s syndrome, Behçet’s disease). In addition, it
can be seen with the genetic disorders Marfan’s syndrome and Ehlers-Danlos syndrome.

Correct Answer. a,b,c,d

(66). Toxicity of MOPP regimen for Hodgkin’s disease includes the following:

a. Sterility

b. Bone hyperplasia

c. Sensory & motor neuropathy

d. Cardiotoxicity

e. Secondary malignancy

Solution. ACE
Explanation: BM depression is seen; not hyperplasia. Also, cardiotoxicity seen in ABVD regimen due to anthracyclines; not in MOPP
regimen. Others are seen.

Copyright © 2014 Delhi Academy of Medical Sciences, All Rights Reserved. 27/110
Correct Answer. a,c,e

(67). One should suspect an acute exacerbation in COPD patients when the patient develops:

a. Increased cough

b. Increased sputum production

c. New onset stridor

d. Disappearance of fever

e. Change in baseline dyspnea

Solution. ABE
Explanation: In a COPD patient, acute exacerbation should be suspected when the patient develops:
- Increased cough or increase in sputum production
- Appearance of fever
- Wheezing
- Change in colour& thickness of sputum
- Tightness in the chest
- Change in the baseline dyspnea

Correct Answer. a,b,e

(68). Not seen in Pickwickian syndrome is/are?

a. Wasting

b. Hyperventilation

c. Ventilation-perfusion mismatch

d. Daytime hypercapnia

e. Leptin resistance

Solution. AB
Explanation:
The obesity hypoventilation syndrome (OHS) / Pickwickian syndrome, is defined by the
1. presence of obesity (body mass index, BMI ≥30 kg/m2),
2. chronic alveolar hypoventilation with daytime hypercapnia (awake PaCO2 ≥45 mm Hg),
3. sleep-related breathing disorder in the absence of any other causes of hypoventilation. The
pathogenesis of hypoventilation in these patients is the result of multiple physiologic variables and conditions
a. including OSA, increased work of breathing,
b. respiratory muscle impairment, ventilation-perfusion mismatching,
c. depressed central ventilatory responsiveness to hypoxemia and hypercapnia, including
“resistance” to leptin (a protein produced mainly by adipose tissue that stimulates ventilation) likely exists.
-The treatment of OHS is similar to that for OSA: weight reduction and nocturnal NIPPV

Correct Answer. a,b

(69). Pre-systolic murmurs may be heard in:

a. Left atrial myxoma

b. MS

c. MR

d. TS

e. TR

Solution. ABD
Explanation: Late diastolic/Presystolic murmurs are heard in MS, TS and Atrial myxomas

Correct Answer. a,b,d

(70). On doing routine USG, an incidental adrenal mass is detected. The following need to be ruled out in the patient:

Copyright © 2014 Delhi Academy of Medical Sciences, All Rights Reserved. 28/110
a. Cushing’s disease

b. Metastasis

c. Adrenal adenoma

d. Carcinoma

e. Adrenal hyperplasia

Solution. ABCDE
Explanation: Incidentaloma is a term used for such entity. Can be Preclinical cushing’s disease,
Pheochromocytoma, Metastasis to adrenals, Hyperplasia, adenoma or carcinoma. Rarely adrenal
cysts may present as incidentaloma.

Correct Answer. a,b,c,d,e

(71). The conditions associated with HCV include:

a. Lichenoid eruptions

b. Celiac disease

c. Glomerulonephritis

d. Gianotti-cronsti syndrome

e. Cryoglobulinemias

Solution. ACE
Explanation: HCV associations include Cryoglobulinemias, MPGN, Lichen planus, Autoimmune
thyroiditis, Monoclonal gammopathies, Vasculitis, NHL, polyarthritis, Idiopathic pulmonary fibrosis

Correct Answer. a,c,e

(72). The hallmark finding seen in Neurofibromatosis-1 include:

a. Neurofibromas

b. Axillary freckling

c. Café-au lait macules

d. Pheochromocytoma

e. Lisch nodules

Solution. C
Explanation: “Café au lait macules are considered the hallmark of Neurofibromatosis-1, seen in
almost 100% patients”.

Correct Answer. c

(73). Beta blockers which improve survival in CHF include:

a. Propranolol

b. Bisoprolol

c. Carvedilol

d. Nebivolol

e. Pindolol

Solution. BC
Explanation: Beta blockers which improve survival in CHF include:
CBM: Carvedilol, Bisoprolol, Metoprolol.
Other beta blockers are useful in other heart diseases, but not recommended in the management of
CHF.

Correct Answer. b,c

Copyright © 2014 Delhi Academy of Medical Sciences, All Rights Reserved. 29/110
(74). SAAG more than 1.1 g/dl but ascitic protein less than 2.5 g/dl is consistent with which among the following conditions?

a. Cirrhosis

b. CCF

c. Nephrotic syndrome

d. TB

e. Massive liver metastasis

Solution. AE
Explanation:

Correct Answer. a,e

(75). A 56-year-old man with a long history of alcohol use is admitted to the hospital after he had a seizure that was witnessed by his wife. The
patient is obtunded, swollen, and jaundiced.
Temperature is 36.8°C (98.3°F), pulse rate is 104/min and regular, respirations are 18/min, and blood pressure is 104/62 mmHg. On
physical examination, auscultation of the chest shows diminished breath sounds. Examination of the abdomen shows firmness of the right
upper quadrant, palpable hepatomegaly, and protuberance of the umbilicus. Results of laboratory studies of serum include the following:
Creatinine 0.8 mg/dL, Sodium 108 mEq/L, Potassium 3.5 mEq/L. Blood urea nitrogen 4 mg/dL. Which of the following is the most
appropriate initial management?

a. Emergent hemodialysis

b. Fluid restriction

c. 3% saline administration

d. 5% dextrose administration

e. Phenobarbitone loading dose

Solution. C
Explanation: The presenting symptom of mental status changes along with the physical examination and laboratory findings indicate that
the seizure was most likely secondary to hyponatremia. More rapid increase in serum sodium level is indicated in patients with
hyponatremia when neurologic manifestations are present. Therefore, the most appropriate initial management of the patient’s condition
is an intervention that will elevate sodium levels — intravenous administration of 3% saline.

Correct Answer. c

(76). All are true about use of triptans in migraine except:

Copyright © 2014 Delhi Academy of Medical Sciences, All Rights Reserved. 30/110
a. Used in prophylaxis of migraine

b. Used in acute treatment of migraine

c. Concomitant use of ergot is always indicated

d. Can be given for long term prophylaxis safely.

e. Given when NSAIDs is not effective

Solution. BE
Explanation: Remember that triptans are useful in acute therapy, but not used in prophylaxis of migraine.

Correct Answer. b,e

(77). Treatment of crohn’s disease includes:

a. Steroids

b. 5-ASA

c. Azathioprine

d. Daclizumab

e. Adalimumab

Solution. ABCE
Explanation: Drugs effective in Crohn’s disease:
- 5-ASA agents e.g. Mesalamine
- Antibiotics
- Corticosteroids
- Anti-TNF alpha therapy, e.g. Infliximab, Adalimumab & Certolizumab
- Immunomodulators, e.g. Azathioprine, 6-MP
- Methotrexate

Correct Answer. a,b,c,e

(78). Cause(s) of acute hyponatremia may include?

a. Thiazide use

b. Post TURP

c. Raised ICP

d. Polydipsia

e. Vomiting

Solution. ABD

Correct Answer. a,b,d

Copyright © 2014 Delhi Academy of Medical Sciences, All Rights Reserved. 31/110
(79). Malignancies associated with secondary polycythemia include(s) all except?

a. RCC

b. Pheochromocytoma

c. Ca breast

d. Meningioma

e. Hepatoma

Solution. C
Explanation: As per Harrison’s latest ed, the following malignancies are associated with secondary
polycythemia:
- Hypernephroma/RCC
- Hepatoma
- Cerebellar hemangioblastoma
- Meningioma
- Pheochromocytoma
- Adrenal tumours
- Uterine fibromyoma

Correct Answer. c

(80). True regarding Vitamin B12-asociated neuropathy?

a. Distal neuropathy with sensory loss

b. Axonal sensorimotor neuropathy

c. Ataxia is present

d. Neuropathy is always associated with hematological changes

e. Can be improved by Vitamin B12 supplements in all patients without sequlae

Solution. ABC
Explanation: Vitamin B12 associated neuropathy
• Complaints of numb hands typically appear before lower extremity paresthesias are noted.
• A preferential large-fiber sensory loss affecting proprioception and vibration with sparing of
small-fiber modalities is present
• An unsteady gait reflects sensory ataxia.
• These features, coupled with diffuse hyperreflexia and absent Achilles reflexes, should
always focus attention on the possibility of cobalamin deficiency.
• Optic atrophy and, in severe cases, behavioural changes ranging from mild irritability and
forgetfulness to severe dementia and frank psychosis may appear. The full clinical picture of
subacute combined degeneration is uncommon. CNS manifestations, especially pyramidal
tract signs, may be missing, and in fact some patients may only exhibit symptoms of
peripheral neuropathy.
• EDx shows an axonal sensorimotor neuropathy. CNS involvement produces abnormal
somatosensory and visual evoked potential latencies. The diagnosis is confirmed by finding
reduced serum cobalamin levels.
• In up to 40% of patients, anemia and macrocytosis are lacking.
• Cobalamin deficiency can be treated with various regimens of cobalamin. One typical
regimen consists of 1000 μg cyanocobalamin IM weekly for 1 month and monthly thereafter.
• An oral cobalamin dose of 1000 μg per day may also be sufficient.
• Treatment for cobalamin deficiency usually does not completely reverse the clinical
manifestations, and at least 50% of patients exhibit some permanent neurologic deficit.

Correct Answer. a,b,c

(81). All of the following are true regarding management of non-ST segment elevation MI (NSTEMI), except?

a. Raised serum troponin levels

b. Management includes the use of aspirin, clopidogrel and LMWH

c. Fibrinolysis is not useful

d. Thrombolysis done within 48 hours

e. Low-risk patient need angiography after 48 to 72 hours to decide management

Copyright © 2014 Delhi Academy of Medical Sciences, All Rights Reserved. 32/110
Solution. D
Explanation: Non-ST segment elevation MI (NSTEMI)
• Patients with NSTEMI have elevated biomarkers of necrosis, such as cardiac troponin I or T,
which are specific, sensitive, and the preferred markers of myocardial necrosis. The MB
isoform of creatine kinase (CK-MB) is a less sensitive alternative. Elevated levels of these
markers distinguish patients with NSTEMI from those with UA.
• Medical therapy involves simultaneous anti-ischemic and antithrombotic treatments and
consideration of coronary revascularization.
• Fibrinolysis is not indicated in NSTEMI.
• ANTI-ISCHEMIC TREATMENT: Nitrates/Beta blockers
• Antithrombotic therapy: Antiplatelet agents like aspirin or clopidogrel, anticoagulants like
LMWH
• Patients at high risk of death undergo angiography within 48 hours, whereas those with lowrisk
undergo angiography after 48 to 72 hours

Correct Answer. d

(82). Platelet aggregation disorder(s) include(s)?

a. vWD

b. Bernard-Soulier syndrome

c. Glanzmann’s thrombasthenia

d. Scott’s syndrome

e. Aspirin use

Solution. C

Correct Answer. c

(83). Not a feature of POEMS syndrome?

a. Polyneuropathy

b. Hepatosplenomegaly

c. Esophageal dysmotility

Copyright © 2014 Delhi Academy of Medical Sciences, All Rights Reserved. 33/110
d. Sensorineural deafness

e. Hirsutism

Solution. CD
Explanation: POEMS syndrome (= Crow-Fukase syndrome) is characterized by
• P: Polyneuropathy
• O: Organomegaly (Hepatosplenomegaly)
• E: Endocrinopathy (DM type II, Hypothyroidism, Amenorrhea, Hyperprolactinemia etc.)
• M: M-protein is elevated
• S: Skin changes (Hyperpigmentation, Hirsutism, Hyperhidrosis & Dermal thickening)

Correct Answer. c,d

(84). A 39 year old obese patient presents with features of UTI, RBS 200 mg% and urine shows pus cells. True statement regarding her therapy
is/are?

a. Glipizide is DOC

b. Insulin can be given

c. Start ciprofloxacin

d. Test for microalbuminuria

e. Start metformin

Solution. C
Explanation: Since nothing more is mentioned, simply age, obesity and blood glucose levels are
insufficient to determine if she is type 1 or type 2 diabetic. However, in any case of diabetes with
UTI, we need to give antibiotics e.g. Ciprofloxacin.

Correct Answer. c

(85). Presentation of Behcet Syndrome may include the following:

a. Non-Erosive arthritis

b. Recurrent aphthous ulceration of the mouth

c. Uveitis

d. Genital Ulcer

e. Pathergy Test

Solution. ABCDE
• Explanation: Behcet Syndrome: A multisystem disorder, diagnosed on the following criteria:
Recurrent oral ulceration PLUS 2 of the following:
- Recurrent genital ulceration
- Eye lesions (Anterior or posterior Uveitis)
- Skin lesions
- Pathergy test
Although not part of diagnostic criteria, Non-erosive arthritis seen in 50% patients.

Correct Answer. a,b,c,d,e

(86). Cause(s) of transudative pleural effusions may include?

a. CCF

b. Cirrhosis

c. TB

d. Churg-Strauss Syndrome

e. SVC obstruction

Copyright © 2014 Delhi Academy of Medical Sciences, All Rights Reserved. 34/110
Solution. ABE

Correct Answer. a,b,e

(87). Factors associated with precipitation of Guillain-barre syndrome (GBS) include?

a. Campylobacter jejuni infection

b. Meningococcal vaccination

c. CMV infection

d. Mycoplasma pneumoniae infection

e. Pneumococcal polysaccharide vaccination

Solution. ACD
Explanation: Approximately 70% of cases of GBS occur 1–3 weeks after an acute infectious process, usually respiratory or
gastrointestinal. Most cases are preceded by infection or reinfection with Campylobacter jejuni. A similar proportion is preceded by a
human herpes virus infection, often CMV or Epstein-Barr virus. Other viruses (e.g., HIV, hepatitis E) and also Mycoplasma pneumoniae
have been identified as agents involved in antecedent infections, as have recent immunizations, including the swine influenza vaccine and
neural rabies vaccine. Meningococcal and pneumococcal vaccines do not increase the risk.

Correct Answer. a,c,d

(88). Which among the following genes are associated with decreased risk of Alzheimer’s disease?

a. Presenilin-1

b. Presenilin-2

c. APP

d. Apo-E2

e. Apo-E4

Solution. D
Explanation: Genetic basis of Alzheimer disease (V.IMP, Recent changes in New Harrison)
Many genes are associated with increased risk of AD, including:
i. APP gene on Ch.21
ii. Presenilin-1 gene on Ch.14
iii. Presenilin-2 gene on Ch.1
Iv. Apo-E gene on Ch.19
However, recent research has revealed that Apo-E gene exists in multiple isoforms. Among them, Apo-E4 is the one which increases risk
of sporadic type of AD. However, Apo-E2 allele has been shown to DECREASE the risk of AD.

Correct Answer. d

(89). Criteria to define Sytemic inflammatory response syndrome(SIRS) includes:

a. Fever> 38 C

b. Hypothermia <36 C

Copyright © 2014 Delhi Academy of Medical Sciences, All Rights Reserved. 35/110
c. Heart rate > 120/min

d. TLC > 15000

e. 30% band cells in periphery

Solution. AB
Explanation: SIRS (Systemic inflammatory response syndrome): Defined to be there when any 2 out of the following are present:
i. Fever >38 C or Hypothermia <36 C
ii. Tachypnea, with RR> 24/min
iii. Tachycardia, with HR>90/min
iv. Leukocytosis (TLC>12000) or Leukopenia (TLC<4000) or Band cells >10%

Correct Answer. a,b

(90). Risk factors in syncope, requiring hospitalization and detailed evaluation include?

a. Prolonged QT interval

b. Grade 2 systolic murmur

c. Brugada pattern on ECG

d. Syncope at rest

e. Syncope on fasting

Solution. ACD

Correct Answer. a,c,d

(91). An AIDS patient presents with confusion, altered sensorium & seizures. The causes can be:

a. Toxoplasmosis

Copyright © 2014 Delhi Academy of Medical Sciences, All Rights Reserved. 36/110
b. PMFL

c. Cryptococcal meningitis

d. AIDS-dementia complex

e. CMV

Solution. ABCDE
Explanation: All the entities can produce acute CNS picture with seizures and altered sensorium

Correct Answer. a,b,c,d,e

(92). Treatment of hyperkalemia include(s)?

a. IV dextrose infusion

b. Parenteral Sodium bicarbonate

c. Parenteral Calcium

d. Albuterol

e. Ipratropium

Solution. ABCD
Explanation:

Correct Answer. a,b,c,d

(93). Staphylococcal proteins involved in attachment of staphylococcus epidermidis to implants include(s)?

a. Autolysins

b. Hemolysins

c. Fibrinogen-binding protein

d. Glycocalyx

e. Lipopolysaccharide

Solution. ACD
Explanation:
S. epidermidis is uniquely adapted to colonize artificial devices like prosthesis and shunts because of its capacity to elaborate the
extracellular polysaccharide (glycocalyx or slime) that facilitates formation of a protective biofilm on the device surface. Implanted
prosthetic material is rapidly coated with host serum or tissue constituents such as fibrinogen or fibronectin. These molecules serve as
potential bridging ligands, facilitating initial bacterial attachment to the device surface. A number of staphylococcal surface-associated
proteins, such as autolysin (AtlE), fibrinogen-binding protein, and accumulation-associated protein (AAP), appear to play a role in
attachment to either modified or unmodified prosthetic surfaces. The polysaccharide intercellular adhesin facilitates subsequent
staphylococcal colonization and accumulation on the device surface.

Correct Answer. a,c,d

(94). Halophilic vibrios include which among the following?

a. Vibrio cholerae

b. Vibrio parahemolyticus

Copyright © 2014 Delhi Academy of Medical Sciences, All Rights Reserved. 37/110
c. Vibrio vulnificus

d. Vibrio alginolyticus

e. Vibrio damsela

Solution. BCDE
Explanation: Species of vibrio requiring 7-10% NaCl for growth are labelled as Halophilic vibrios. They include Vibrio parahemolyticus,
vibrio vulnificus, vibrio alginolyticus, vibrio damsela and vibrio fluvialis. Growth on CLED medium may be used to differentiate them from
non-halophilic vibrios

Correct Answer. b,c,d,e

(95). Not true regarding leptospirosis is/are?

a. Traveller’s disease

b. A zoonosis

c. Both epidemic and endemics occur

d. Transmission can not occur across conjunctiva

e. Complement mediated killing protects against leptospira infection

Solution. DE
Explanation:
• Leptospirosis has a worldwide distribution but occurs most commonly in the tropics and
subtropics because the climate and occasionally poor hygienic conditions favour the
pathogen’s survival and distribution.
• As a zoonosis, leptospirosis affects almost all mammalian species and represents a
significant veterinary burden. Rodents, especially rats, are the most important reservoir,
although other wild mammals as well as domestic and farm animals may also harbour these
microorganisms.
• Leptospirosis presents as both an endemic and an epidemic disease. Transmission of
leptospires may follow direct contact with urine, blood, or tissue from an infected animal or,
more commonly, exposure to environmental contamination.
• In addition, leptospirosis is a traveller’s disease. Large proportions of patients acquire the
infection while traveling in tropical countries, usually during adventurous activities such as
white-water rafting, jungle trekking, and caving. Transmission occurs through cuts, abraded
skin, or mucous membranes, especially the conjunctival and oral mucosa.
• The organisms are able to survive in the nonimmune host: they evade complementmediated
killing by binding factor H, a strong inhibitor of the complement system, on their
surface. Moreover, pathogenic leptospires resist ingestion and killing by neutrophils,
monocytes, and macrophages.

Correct Answer. d,e

(96). Autoclaving is done for?

a. Glass

b. Bronchoscope

c. Metallic objects

d. Plastics

e. Wooden materials

Copyright © 2014 Delhi Academy of Medical Sciences, All Rights Reserved. 38/110
Solution. C

Correct Answer. c

(97). Causative agents of gas gangrene include?

a. Clostridium tetani

b. Clostridium novyi

c. Chistolyticum

d. Clostridium septicum

e. Clostridium tertium

Solution. BCDE
Explanation: Histotoxic clostridial species such as C. perfringens, C. histolyticum, C. septicum, C. novyi, and C. sordelliicause aggressive
necrotizing infections of the skin and soft tissues, principally gas gangrene. Sometimes, clostridium tertium also causes sponatneous gas
gangrene.

Correct Answer. b,c,d,e

(98). Features which distinguish Actinomycosis from Nocardia is/are:

a. Grow at wide range of temperature

b. Environmental pathogen

c. Not acid fast positive

d. Endogenous cause of disease

e. Facultative anaerobes

Copyright © 2014 Delhi Academy of Medical Sciences, All Rights Reserved. 39/110
Solution. CDE

Correct Answer. c,d,e

(99). Which among the following is FALSE regarding Legionella?

a. No man to man transmission is seen

b. Aspiration is the MC method of transmission

c. Causes Pontiac fever

d. Microaerophilic Gram positive bacilli

e. Hyponatremia is seen

Solution. D
Explanation: Legionella is an aerobic, Gram-negative, non-encapsulated bacilli.
• Outbreaks of Legionella infection are associated with Contaminated water sources such as
AC cooling plants
• Multiple modes of transmission: Aspiration (MC), Aerosolization, Direct Instillation, etc.
• No Man to man transmission
• Manifestations: Can cause Pneumonia(Legionnaire’s Disease) or Pontiac fever (Self-limited)
• Hyponatremia may be seen
• BCYE agar is the media of choice for culture

Correct Answer. d

(100). Which among the following is/are reasons responsible for false-positive VDRL test?

a. Autoimmune diseases

b. Drug abuse

c. Leprosy

d. Hepatitis

e. Lyme disease

Copyright © 2014 Delhi Academy of Medical Sciences, All Rights Reserved. 40/110
Solution. ABC

Correct Answer. a,b,c

(101). Not a correct match for E. coli and its disease?

a. ETEC: Traveller’s diarrhea

b. EPEC: Affects mainly adults

c. EHEC: Bloody diarrhea

d. EAEC: Prolonged diarrhea in immunosuppressed

e. EIEC: Neonatal diarrhea

Copyright © 2014 Delhi Academy of Medical Sciences, All Rights Reserved. 41/110
Solution. BE
Explanation:

Correct Answer. b,e

(102). True statements is/are:

a. Kaposi’s sarcoma is caused by HHV-6

Copyright © 2014 Delhi Academy of Medical Sciences, All Rights Reserved. 42/110
b. HTLV causes Tropical spastic paraparesis

c. Bronchiolitis is caused by RSV

d. Erythema infectiosum has ‘slapped cheek appearance’

e. Roseola infantum is caused by Parvovirus

Solution. BCD
Explanation: Kaposi sarcoma is associated with HHV-8
Roseola infantum/ sixth disease is due to HHV-6
Erythema infectiosum is caused by Parvovirus B19; produces slapped cheek appearance

Correct Answer. b,c,d

(103). MURCS syndrome includes?

a. Mullerian agenesis

b. Horse-shoekindeys

c. SNHL

d. Mental retardation

e. TAPVC

Solution. ABC
Explanation: MURCS SYNDROME
• MU(mullerian) R(renal) C(cervicothoracic) S(somite) abnormalities
This is the special name given to Triad of mullerian, renal and skeletal abnormalities and is
also called as MRKH type 2 syndrome also called as atypical MRKH
• Apart from mullerian agenesis associated malformations seen in MRKH type 2 include upper
urinary tract malformations (40% of cases), unilateral renal agenesis (23-28%) ectopia of one
or both kidneys (17%), renal hypoplasia (4%), horseshoe kidney and hydronephrosis.
• Skeletal abnormalities, mainly of the spine and less frequently in the face and limbs, are also
reported. Spinal malformations encountered include scoliosis, isolated vertebral anomalies.
Limb abnormalities include duplicated thumb and absent radius
• Hearing impairment, due to middle ear malformations or sensorineural defects, is seen in
10-25% of cases. Heart malformations are very rare but include valvular pulmonary stenosis,
Tetralogy of Fallot

Correct Answer. a,b,c

(104). Biophysical profile scoring includes:

a. Fetal muscle tone

b. Fetal breathing movements

c. Pulsatility index

d. Amniotic fluid volume

e. Doppler velocimetry of umbilical artery

Solution. ABD
Explanation: Manning’s biophysical profile scoring (Observe for 30 min)
Normal score=2, Abnormal score=0.
Includes 5 parameters:
i. Non stress test (NST)
ii. Fetal breathing movements
iii. Gross body movements
iv. Fetal muscle tone
v. Amniotic fluid pockets

Correct Answer. a,b,d

(105). A healthy 20-year-old G1P0 presents for her first OB visit at 10 weeks gestational age. She denies any significant medical history both
personally and in her family. All of the following tests should be ordered as part of the initial prenatal care visit

a. CBC

Copyright © 2014 Delhi Academy of Medical Sciences, All Rights Reserved. 43/110
b. HIV

c. Hepatitis B surface antigen

d. Type and screen

e. Two-hour glucose tolerance test

Solution. ABCDE
Explanation: A 1-h glucose tolerance test should be performed between 24 and 28 weeks for women at risk for gestational diabetes. It is
recommended that all women undergo tests for hepatitis B, HIV, type and screen, and CBC at the first prenatal visit

Correct Answer. a,b,c,d,e

(106). Pre-requisites for forceps delivery are all except-

a. Fully dilated cervix

b. Unruptured membranes

c. No obstruction to vaginal delivery

d. Engaged head

e. Contracted pelvis

Solution. BE
Explanation: Prerequisites for forceps delivery include the following:
• The head must be engaged.
• The cervix must be fully dilated and retracted.
• The position of the head must be known.
• The type of pelvis should be known.
• The membranes must be ruptured.
• No disproportion should be suspected between the size of the head and the size of the
pelvic inlet and mid pelvis.
• The patient must have adequate anesthesia.
• Adequate facilities and supportive elements should be available.
• The operator should be fully competent in the use of the instruments and the recognition
and management of potential complications.
• An operator should be present who knows when to stop, to not force the issue, and to not
aggressively use both forceps and vacuum in combination because this has been shown to
increase morbidity for both the mother and fetus.

Correct Answer. b,e

(107). Which is not included in amsels criteria

a. pH>4.5

b. Presence of fishy odour on 10% KOH

c. Severe itching

d. Small bacteria coating epithelial cells seen

e. Presence of polymorphs

Solution. CE
Explanation: Amsels criteria for bacterial vaginosis includes-
Any 3 of -
1. Thin homogenous greyish white discharge
2. Wet film showing small bacterial coatinf epithelial cells called clue cells
3.10% KOH mount releasing fishy odour- whiff test.
4. pH >4.5

Correct Answer. c,e

(108). Benefits of 2nd trimester USG include all except:

a. Fetal viability

b. Detection of congenital anomalies

Copyright © 2014 Delhi Academy of Medical Sciences, All Rights Reserved. 44/110
c. Placental localization

d. Adjunct to amniocentesis

e. Fetal biometry

Solution. A
nd
Explanation: Benefits of 2 trimester USG include:
- Detection of multiple pregnancy
- Detection of congenital anomalies
- Placental localization
- Monitoring of high risk pregnancy
- Baseline record of fetal biometry
- Adjunct to procedures like amniocentesis, cordocentesis, cervical enciclage, ECV etc.

Correct Answer. a

(109). True about vaccum extraction of fetus is/are?

a. Can be used in non-dilated cervix

b. Can be used in incompletely dilated cervix

c. Used in face presentation

d. Applied 3 cm posterior to anterior fontanelle

e. Applied 3 cm anterior to posterior fontanelle

Solution. BE
Explanation: Ventouse can be used in incompletely dilated cervix. The center of the cup is applied
over the saggital suture and about 3 cm in front of the posterior fontanelle. Face presentation is a
contraindication for ventouse application.

Correct Answer. b,e

(110). Stage III of the carcinoma endometrium comprises?

a. Vagina involvement

b. Adnexal extension

c. Bladder involvement

d. Bowel involvement

e. Inguinal lymph node involvement

Copyright © 2014 Delhi Academy of Medical Sciences, All Rights Reserved. 45/110
Solution. AB
Explanation:

Correct Answer. a,b

(111). Clinical picture of premenstrual tension may include?

a. Behavioural abnormalities

b. Cardiac failure

c. Bloating

d. Abdominal cramps

e. Poor response to NSAIDs

Solution. ACD
Explanation:

Correct Answer. a,c,d

(112). True regarding Swyer’s syndrome is/are?

a. Pure gonadal dysgenesis

Copyright © 2014 Delhi Academy of Medical Sciences, All Rights Reserved. 46/110
b. Female hermaphrodite

c. Oversecretion of MIF

d. No response to external estrogen

e. Increased risk of gonadal malignancy

Solution. AE
Explanation: Swyer’s syndrome
• This syndrome is a male pseudohermaphrodite, a pure 46 XY gonadal dysgenesis with
presence of uterus and the cervix but with hypo-oestrogenism and poorly developed
breasts.
• Undeveloped testes do not secrete testosterone and MIF resulting in the development of
female genital organs and female phenotype.
• The woman presents with primary amenorrhoea, absence of secondary sex characters and
female external genitalia.
• Cyclical oestrogen and progestogen can induce menstruation. Conception with in vitro
fertilization (IVF) using donor eggs is a possibility.
• The gonads (testis) have 30% risk to undergo malignancy and should be removed.

Correct Answer. a,e

(113). Indications for minimal invasive surgery in tubo-ovarian abscess is/are?

a. Abscess size less than 5 cm

b. Pyoperitoneum

c. No response to broad spectrum antibiotics for >48 hr

d. Ruptured abscess

e. High grade fever

Solution. BCD
Explanation: Minimal invasive surgery is required in TOA if:
• The size of the abscess is more than 10 cm.
• The abscess fails to respond to antibiotics in 48–72 h.
• Abscess ruptures.
• Pyoperitoneum

Correct Answer. b,c,d

(114). Absolute Contraindications for IUCD use include(s)?

a. Multiparity

b. Septate uterus

c. Severe anemia

d. Previous ectopic pregnancy

e. Fibroids

Solution. BC
Explanation: Contraindications for IUCD use include
• Suspected pregnancy
• Pelvic inflammatory disease (PID), lower genital tract infection
• Presence of fibroids—because of misfit n Menorrhagia and dysmenorrhoea, if Copper T is
used
• Severe anaemia
• Diabetic women who are not well controlled—because of slight increase in pelvic infection
• Heart disease—risk of infection
• Previous ectopic pregnancy
• Scarred uterus
• Preferably avoid its use in unmarried and nulliparous patients because of the risk of PID and
subsequent tubal infertility
• LNG IUCD in breast cancer
• Abnormally shaped uterus, septate uterus

Correct Answer. b,c,e

Copyright © 2014 Delhi Academy of Medical Sciences, All Rights Reserved. 47/110
(115). Which among the following are True statements regarding Androgen Insensitivity Syndrome?

a. Normal Breast development

b. Short blind vaginal pouch

c. Well-developed pubic hair

d. X linked dominant condition

e. Testis is hypoplastic

Solution. AB
Explanation: Androgen Insensitivity Syndrome (=Testicular Feminization Syndrome) is a condition which may be X-linked recessive as
well as sporadic. Karyotype is XY but phenotypically they are females. They have Normal breast development, normal testis (but
rd
abnormally placed), primary amenorrhea, absent uterus, fallopian tubes and upper 2/3 vagina. A short blind vaginal pouch may be seen.
Pubic and axillary hair is absent. Gonadectomy and hormonal replacement is the mainstay
of treatment.

Correct Answer. a,b

(116). Combined OCPs increase the risk of which among the following?

a. Endometrial carcinoma

b. Hirsutism

c. Benign breast disease

d. Carcinoma cervix

e. Venous thromboembolism

Solution. DE
Explanation:Non-contraceptive benfits of OCPs
- Decreased risk of
 - Ovarian cancer
 - Endometrial cancer
 - Benign breast disease
 - Functional ovarian cysts
 - Ectopic pregnancy
 - Osteopenia and osteoporotic fractures
- Useful in the management of PID, acne, hirsutism and dysmenorrhea
Also note the following risks of OCPs as per Harrison

Copyright © 2014 Delhi Academy of Medical Sciences, All Rights Reserved. 48/110
Correct Answer. d,e

(117). High mortality in neonate is seen if pregnant female gets chicken pox rash around what time?

a. 1st trimester
nd
b. 2 trimester
rd
c. 3 trimester

d. Near delivery

e. No mortality seen

Solution. D
Explanation: Mortality is particularly high in neonates born to susceptible motherswho contracted varicella around the time of delivery.
Infants whosemothers demonstrate varicella in the period from 5 days prior to deliveryto 2 days afterward are at high risk for severe
varicella. These infantsacquire the infection transplacentally as a result of maternal viremia,which may occur up to 48 hr prior to onset of
maternal rash. The syndrome comprises rash and pneumonia and is called neonatal varicella syndrome. Needs VZIG and acyclovir within
12 hr of birth.

Correct Answer. d

(118). True about outcome of division after fertilization?

a. Division before 72 hr results in monoamniotic monochorionic twins

b. Division before 72 hr results in diamniotic dichorionic twins

c. Division between 5 to 8 days produces monoamniotic monochorionic twins

d. Division after embryonic disc formation produces conjoint twins

e. Division at 14 days produces Siamese or conjoint twins

Copyright © 2014 Delhi Academy of Medical Sciences, All Rights Reserved. 49/110
Solution. BDE
Explanation: Mechanism of twin pregnancy
- Dizygotic fertilization of 2 separate ova by 2 sperms produces: Dizygotic twins
- Monozygotic splitting of single fertilized egg produces: Monozygotic twins
- Division within 72 hr (at morula stage): Dichorionic diamnotic
- Division between 4-7 days (Blastocyst): Monochorionic diamniotic
- Division between 8-13 days (After implantation): Monochorionic monoamniotic
- Division >13 days (after embryonic disc formation): Conjoined/Siamese twins

Correct Answer. b,d,e

(119). Supranuclear vertical gaze abnormalities can be seen in all of the following conditions except:

a. Myasthenia gravis

b. Parkinson disease

c. Pineal region tumours

d. Ataxia–telangiectasia

e. Steel Richardson syndrome

Solution. A
Explanation: Myasthenia gravis might produce ophthalmoplegia that mimics supranuclear vertical gaze abnormalities, but the
pathophysiology is at the ACh receptor. Parkinson disease, pineal region tumors (dorsal midbrain syndrome), and ataxia–telangiectasia all
can cause supranuclear gaze palsy.

Correct Answer. a

Copyright © 2014 Delhi Academy of Medical Sciences, All Rights Reserved. 50/110
(120). Sequelae of corneal perforation may include?

a. Iris prolapse

b. Panophthalmitis

c. Intraocular hemorrhage

d. Choroidal melanoma

e. Vitreous hemorrhage

Solution. ABCE
Explanation: Sequelae of corneal perforation include:
i. Prolapse of iris. It occurs immediately followingperforation in a bid to plug it.
ii. Subluxation or anterior dislocation of lensmay occur due to sudden stretching and ruptureof
zonules.
iii. Anterior capsular cataract. It is formed whenthe lens comes in contact with the ulcer following
a perforation in the pupillary area.
iv. Corneal fistula. It is formed when the perforationin the pupillary area is not plugged by iris and
is lined by epithelium which gives wayrepeatedly. There occurs continuous leak ofaqueous through
the fistula.
v. Purulent uveitis, endophthalmitis or evenpanophthalmitismay develop due to spread ofintraocular
infection.
vi. Intraocular haemorrhage in the form of eithervitreous haemorrhage or expulsive
choroidalhaemorrhage may occur in some patients due tosudden lowering of intraocular pressure

Correct Answer. a,b,c,e

(121). True regarding Acanthamoeba kaeratitis in eye?

a. More in those wearing rigid gas permeable than soft contact lenses

b. More in those wearing hard contact lens than soft contact lens

c. Pain is out of proportion to clinical signs

d. Opportunistic infection in herpetic keratitis

e. Acanthamoeba can be cultured on E.coli enriched agar

Solution. CDE
Explanation: Acanthamoeba keratitis
• Occurs due to direct corneal contact with contaminated water having acanthamoeba
• More common in soft contact lens wearers (especially those using home-made saline)
• Can also occur as an opportunistic infection in those with herpetic keratitis, bacterial
keratitis, bullous keratopathy and neuroparalytic keratitis
• Clinically presents with severe pain (out of proportion to degree of inflammation), watering,
photophobia, blepharospasm and blurred vision
• Limbitis, fine epithelial opacities and radial kerato-neuritis may be seen, along the corneal
nerves. Hypopyon and central ring-shaped lesion in advanced cases
• Lab diagnosis: Culture on E.coli enriched non nutrient agar shows trophozoites within 48 hrs,
which gradually turn into cysts
• Treatment is unsatisfactory. May include Polymyxin B, Ketoconazole, Voriconazole and
neomycin. Corneal perforation and corneal scarring may need surgery

Correct Answer. c,d,e

(122). Granulomatous uveitis is seen in?

a. Vogt-Koyanagi-Harada disease

b. Fuch’s Disease

c. Behcet’s syndrome

d. Sarcoidosis

e. Psoriasis

Solution. AD
Explanation: Granulomatous uveitis is a chronic, Proliferative inflammation seen in conditions like TB, Leprosy, syphilis, brucellosis,
leptospirosis, Sarcoidosis, VKH syndrome, sympathetic ophthalmitis & even parasitic infections.

Copyright © 2014 Delhi Academy of Medical Sciences, All Rights Reserved. 51/110
Correct Answer. a,d

(123). Not a type of periocular injection technique?

a. Sub-tenon

b. Retrobulbar

c. Intrabulbar

d. Peribulbar

e. Subconjunctival

Solution. C
Explanation: Periocular injections are frequently employed to deliver drugs. These include:
(a) Subconjunctival injections. These are commonly used to achieve higher concentration of
drugs. Further, the drugs which cannot penetrate the cornea owing to large-sized molecules
can easily pass through the sclera.
(b) Sub-Tenon injections. These are preferred over subconjunctival injection. Anterior sub-
Tenon injections are used mainly to administer steroids in the treatment of severe or
resistant anterior uveitis. Posterior sub-Tenon injections are indicated in patients with
intermediate and posterior uveitis.
(c) Retrobulbar injections. These are used to deliver drugs for optic neuritis, papillitis, posterior
uveitis and also for administering retrobulbar block anaesthesia.
(d) Peribulbar injections. These are now frequently used for injecting anaesthetic agents.

Correct Answer. c

(124). Advantages of phacoemulsification may include?

a. Can be done in surface anesthesia

b. Small incision size

c. Better in single eye patient

d. Less cost

e. Less skills required

Solution. AB
Explanation: Merits of phacoemulsification over manual SICS
1. Topical anaesthesia may be sufficient forphacoemulsification in expert hands.
2. Postoperative congestion is minimal afterphacoemulsification, as phaco is usuallyperformed
through a clear corneal incision.
3. Small incision. The chief advantage ofphacoemulsification over manual SICS is that itcan be
performed through a smaller (3.2 mm)incision.
4. Less corneal complications. Phacoemulsificationcan be performed in the posterior
chamberwithout prolapsing the nucleus into the anteriorchamber, thereby minimising the risk of cornealcomplications.
5. Visual rehablitationis comparetively quicker inphacoemulsification as compared to manual SICS.
6. Postoperative astigmatism is comparatively lesswhen foldable IOLs are implanted through
asmaller incision (3.2 mm).

Correct Answer. a,b

(125). Gradual painless loss of vision is seen in:

a. CRAO

b. Corneal ulceration

c. Optic neuritis

d. Optic atrophy

e. Retinitis pigmentosa

Copyright © 2014 Delhi Academy of Medical Sciences, All Rights Reserved. 52/110
Solution. DE
Explanation: Gradual painless loss of vision:
- Corneal dystrophy
- Corneal degeneration
- Cataract
- Chorio-retinal degeneration
- Dry type-ARMD
- Diabetic retinopathy
- Retinitis pigmentosa
- Optic atrophy
- Refractive errors
- Presbyopia

Correct Answer. d,e

(126). Causes of deep anterior chamber include:

a. Keratoconus

b. Keratoglobus

c. Aphakia

d. Globe perforation posteriorly

e. Myopia

Solution. ABCDE
Explanation: Causes of deep anterior chamber include:
- Aphakia
- Total posterior synechiae
- Myopia
- Keratoglobus
- Bupthalmos
- Keratoconus
- Anterior dislocation of lens into the anterior chamber
- Posterior perforation of the globe

Correct Answer. a,b,c,d,e

(127). Aphakia may result in which among the following?

a. Myopia

b. Loss of vision

c. Hypermetropia

d. Increased power of the eye

e. Loss of accommodation

Solution. CE
Explanation: Optics of aphakic eye
Following optical changes occur after removal ofcrystalline lens:
1. Eye becomes highly hypermetropic.
2. Total power of eye is reduced to about +44 Dfrom +60 D.
3. The anterior focal point becomes 23.2 mm in frontof the cornea.
4. The posterior focal point is about 31 mm behindthe cornea i.e., about 7 mm behind the
eyeball.(The antero-posterior length of eyeball is about24 mm)
5. There occurs total loss of accommodation

Correct Answer. c,e

(128). All of the following are indications to perform pan-retinal photocoagulation (PRP) except?

a. Low risk PDR

b. High risk PDR

c. Iris neovascularization

d. Severe NPDR with renal failure

Copyright © 2014 Delhi Academy of Medical Sciences, All Rights Reserved. 53/110
e. Severe NPDR with hepatic failure

Solution. AE
Explanation: Indications for PRP are:
• PDR with high risk characteristics (HRCs),
• Neovascularization of iris (NVI),
• Severe NPDR associated with:
– Poor compliance for follow up,
– Before cataract surgery/YAG capsulotomy,
– Renal failure,
– One-eyed patient, and
– Pregnancy

Correct Answer. a,e

(129). Not true regarding childhood clubfoot?

a. Treatment should be started after 1 year of age

b. Ponseti method is the standard therapy in clubfoot management

c. Hindfoot equinus difficult to correct

d. Tenotomy done as outpatient procedure

e. Arthrodesis done in older children and adults

Solution. A
Explanation: In clubfoot, non-operative treatment is initiated in all infants and should be startedas
soon as possible following birth.ThePonseti methodof clubfoot treatment, which has now become
the standard of initialtreatment, involves a specific technique for manipulation and serialcasting and
may be best described as minimally invasive rather thannonoperative. The order of correction
follows the mnemonic CAVE.Weekly cast changes are performed; 5-10 casts are typically
required.The most difficult deformity to correct is the hindfoot equinus, andapproximately 90% of
patients will require a percutaneous tenotomyof the heel cord as an outpatient. Following the
tenotomy, a long legcast with the foot in maximal abduction (up to 70 degrees) and dorsiflexionis
worn for 3-4 wk; the patient then begins a bracing program.Triple arthrodesis is reserved as salvage
for painful, deformed feet inadolescents and adults.

Correct Answer. a

(130). What is true regarding use of bone scans in acute osteomyelitis?

a. Not useful in adults

b. Increased osteoblastic activity detected

c. 99mTc used for bone scan

d. Low sensitivity in acute osteomyelitis

e. Takes 72 hours after onset of symptoms to be of value in diagnosis

Solution. BC
Explanation: Bone scan in acute osteomyelitis
Radionuclide imaging or bone scan can be valuable in suspected bone infections,especially early in the course of infection and/or if
multiple foci aresuspected or an unusual site is suspected, as in the pelvis. Technetium-99methylene diphosphonate (99mTc), which
accumulates in areas ofincreased bone turnover, is the preferred agent for radionuclide boneimaging (3-phase bone scan). Osteomyelitis
causes increased vascularity,inflammation, and increased osteoblastic activity, resulting in anincreased concentration of 99mTc. Any
areas of increased blood flow orinflammation can cause increased uptake of 99mTc in the 1st and 2ndphases, but osteomyelitis causes
increased uptake of 99mTc in the 3rdphase (4-6 hr). Three-phase imaging with 99mTc has excellent sensitivity (84-100%) and specificity
(70-96%) in hematogenous osteomyelitisand can detect osteomyelitis within 24-48 hr after onset of symptoms.

Correct Answer. b,c

(131). In long bone #, fixation is done with:

a. Intramedullary nails

b. Compression plate

c. External fixation

Copyright © 2014 Delhi Academy of Medical Sciences, All Rights Reserved. 54/110
d. Screws

e. Tension band wiring

Solution. ABCD
Explanation: General management of #
In long bones:
- Intramedullary nailing
- Plating
- External fixators
- Dynamic hip screw (DHS) & Dynamic condylar screw(DCS)
- Plaster cast
In short bones: Screws/K-wires/Tension-band wiring

Correct Answer. a,b,c,d

(132). A man presents with secondary aneurysmal bone cyst in Ortho OPD. What all are the underlyingn conditions you might suspect in that
patient?

a. Osteoclastoma

b. Chondroblastoma

c. Fibrous dysplasia

d. Multiple myeloma

e. Chronic osteomyelitis

Solution. ABC
Explanation: Secondary aneurysmal bone cyst may arise from:
- Osteoclastoma
- Chondroblastoma
- Hemangioendothelioma
- Fibrous dysplasia
- Osteoblastoma
- Osteosarcoma
- Chondromyxoid fibroma
- Non-ossifying fibroma
- Metastatic tumours

Correct Answer. a,b,c

(133). True statement(s) regarding Ewing’s sarcoma is/are?

a. Metaphysis in long bones is common site

b. Primary therapy is surgical

c. Vascular origin

d. Highly radiosensitive tumour

e. Common primary malignant bone tumour in children and adolescents, next only to osteosarcoma

Solution. DE
Explanation: Quick review of Ewing’s sarcoma
• Highly malignant, aggressive bone tumour
• Common in young age: between 10-20 years of age
• Second only to osetosarcoma in frequency in children and adolescents
• MC site: Diaphysis of long bones
• Site of origin: Endothelial cells of the bone marrow
• MC translocation seen: t(11;22)
• MIC-2 (CD99) staining is positive
• Pain, swelling and limited movements are present
• Often associated with constitutional symptoms like fever and weight loss
• X-ray: Onion-peel appearance due to periosteal reaction
• Highly radiosensitive
• Treatment: Radio + chemotherapy

Correct Answer. d,e

(134). Which among the following are common causes of limb length discrepancy?

Copyright © 2014 Delhi Academy of Medical Sciences, All Rights Reserved. 55/110
a. Osteoarthritis

b. Hip fracture

c. Hip dislocation

d. Avascular necrosis

e. Fixed flexion deformity

Solution. ABCDE
Explanation: All the options mentioned are common causes.

Correct Answer. a,b,c,d,e

(135). Which of the following are adverse prognostic indicators in chronic lymphocytic leukemia/small lymphocytic lymphoma?

a. CD 38 positive

b. CD 49d negative

c. ZAP 70 positive

d. Deletion 13q

e. Deletion 11q

Solution. ACE

Correct Answer. a,c,e

(136). CD 20 positivity can be seen in which of the following tumors?

a. Diffuse large B cell lymphoma

b. Follicular lymphoma

c. Mycoses fungoides

d. Anaplastic large cell lymphoma

e. Langerhan cell histiocytosis

Solution. AB
CD 20 is a B cell marker
C and D are T cell lymphomas
E is a dendritic cell tumor

Correct Answer. a,b

(137). Pathogenic T lymphocyte response is implicated in pathogenesis of which of the following vasculitis

a. Giant cell arteritis

b. Takayasu arteritis

c. Polyarteritis nodosa

d. Henoch Schonlein purpura

e. Granulomatosis with polyangiitis

Copyright © 2014 Delhi Academy of Medical Sciences, All Rights Reserved. 56/110
Solution. ABE
Reference: Harrison’s Principles of Internal Medicine, 20th edition

Correct Answer. a,b,e

(138). All of the following are core criteria included in prognostic evaluation of neuroblastoma except

a. Age

b. Evidence of Schwannia stroma

c. Mitotic karyorrhectic index

d. TRKA expression

e. Chromosome 1p loss

Copyright © 2014 Delhi Academy of Medical Sciences, All Rights Reserved. 57/110
Solution. DE
Reference: Robbins and Cotran’s Pathological Basis of Disease, 9th edition

Correct Answer. d,e

(139). Which of the following are histological variants of lung adenocarcinoma as per latest WHO classification?

a. Basaloid

b. Lepidic

c. Enteric

d. Foetal

e. Micropapillary

Copyright © 2014 Delhi Academy of Medical Sciences, All Rights Reserved. 58/110
Solution. BCDE

Correct Answer. b,c,d,e

(140). Which of the following are true about Kawasaki disease

a. Strawberry tongue

b. Acute febrile illness

c. Periungual peeling

d. Small vessel vasculitis

e. Anti endothelial bodies can be seen

Solution. ABCE
Previously called mucocutaneous lymph node syndrome or infantile polyarteritis nodosa, KD is the
second most common vasculitis of childhood with 80% patients being younger than 4 years of age.
Erythema of palms, soles with periungual peeling is seen. Conjunctival and oral erythema is also
present. Cervical lymphadenopathy with an acute febrile illness is characteristic. It is a medium
vessel vasculitis with fibrinoid necrosis and rarely granulomas, with transmural inflammation.
Coronary arteries are the most common vessels involved. Anti endothelial antibodies have been
seen in KD.

Correct Answer. a,b,c,e

(141). Which of the following is/are about medullary carcinoma of breast

a. Heavy mononuclear infiltrate

Copyright © 2014 Delhi Academy of Medical Sciences, All Rights Reserved. 59/110
b. Syncytial growth pattern

c. Minimal or no desmoplastic reaction

d. Associated with BRCA1 mutations

e. Triple negative

Solution. ABCDE
Carcinomas with medullary features are a special typeof triple-negative cancer comprising about 5%
of all breast cancers. These carcinomas typically grow as rounded masses that can be difficult to
distinguish from benign tumors on imaging. They consist of sheets of large anaplastic cells associated
with pronounced lymphocytic infiltrates composed predominantly of T cells. The presence of
lymphocytes is associated with a favorable prognosis, at least in part due to a better response to
chemotherapy compared to poorly differentiated carcinomas without lymphoid infiltrates. It is seen
frequently in women with germline BRCA1 mutations.

Correct Answer. a,b,c,d,e

(142). Which of the following genes are classically associated with follicular carcinoma of thyroid

a. PAX8

b. KRAS

c. BRAF

d. RET

e. PTEN

Copyright © 2014 Delhi Academy of Medical Sciences, All Rights Reserved. 60/110
Solution. ABE

Correct Answer. a,b,e

(143). Which of the following are pro apoptotic?

a. BCL 2

b. BCL XL

c. BCL XS

d. Smac

e. BAX

Solution. CDE
Pro apoptotic molecules include- BAX, BAK, Smac, Diablo, BCL XS, Cytochrome c, APAF1
Anti apoptotic molecules include- BCL2, BCL XL, MCL1, IAPs

Correct Answer. c,d,e

(144). All are examples of type II hypersensitivity reactions except

a. Autoimmune hemolyticanemia

b. Immune thrombocytopenic purpura

Copyright © 2014 Delhi Academy of Medical Sciences, All Rights Reserved. 61/110
c. Pemphigus vulgaris

d. Post streptococcal glomerulonephritis

e. Rheumatoid arthritis

Solution. DE
Examples of type II hypersensitivity

Correct Answer. d,e

(145). Chromogranin and synaptophysin positivity can be seen in which of the following tumors

a. Small cell carcinoma lung

b. Ileal carcinoid

c. Pheochromocytoma

d. Merkel cell carcinoma

e. Ewing sarcoma

Solution. ABCD
- Granin protein located in secretory vesicles of neurons and endocrine cells
- Neuroendocrine and ganglion cell tumors (carcinoid, Merkel cell carcinoma-lung,
neuroblastoma, neuroendocrine carcinoma, paraganglioma, small cell carcinoma),
neuroendocrine hyperplasia)
- Commonly used neuroendocrine marker (also synaptophysin and CD56) for normal cells and
neuroendocrine tumors
- Helps differentiate pheochromocytoma (almost always positive) from adrenocortical
carcinoma (almost always negative)

Correct Answer. a,b,c,d

Copyright © 2014 Delhi Academy of Medical Sciences, All Rights Reserved. 62/110
(146). Granular pattern of immunofluorescence can be seen in which of the following glomerular disorders?

a. Rapidly progressive glomerulonephritis type III

b. Goodpasture syndrome

c. Systemic lupus erythematosus

d. Membranous nephropathy

e. Post streptococcal glomerulonephritis

Solution. CDE
A- ANCA mediated, hence pauci immune
B- Anti GBM mediated, hence linear pattern
C, D, E- immune complex deposition mediated, hence granular pattern

Correct Answer. c,d,e

(147). Which of the following histologies would classify meningioma as grade II or atypical?

a. Secretory

b. Transitional

c. Clear cell

d. Papillary

e. Chordoid

Solution. CE
Atypical meningiomas (WHO grade II/IV) are lesions with a
higher rate of recurrence and more aggressive local growth, and may require radiation therapy in addition to surgery. They are
distinguished from lower grade meningiomas by having four or more mitoses per 10 high power fields or at least three atypical features
(increased cellularity, small cells
with a high nuclearto-cytoplasmic ratio, prominent nucleoli, patternless growth, or necrosis). Certain
histologic patterns (clear cell and chordoid) are also considered to be grade II/IV because of their
more aggressive behaviour.

Correct Answer. c,e

(148). All are true about lymphoplasmacytic lymphoma except

a. IgM is the most commonly secreted Ig

b. Light chains in excess can cause renal failure

c. Hyperviscosity is seen

d. Myd88 mutations are common

e. Bone destruction is common

Solution. BE
Lymphoplasmacytic lymphoma is a B-cell neoplasm of older adults that usually presents in the sixth
or seventh decade of life. Although bearing a superficial resemblance to CLL/SLL, it differs in that a
substantial fraction of the tumor cells undergo terminal differentiation to plasma cells. Most
commonly, the plasma cell component secretes monoclonal IgM, often in amounts sufficient to
cause a hyperviscosity syndrome known as Waldenström macroglobulinemia. Unlike multiple
myeloma, complications stemming from the secretion of free light chains (e.g., renal failure and
amyloidosis) are relatively rare and bone destruction does not occur. Recent deep sequencing
studies have shown that virtually all cases of lymphoplasmacytic lymphoma are associated with
acquired mutations in MYD88

Correct Answer. b,e

(149). Which of the following AML types is/are diagnosed irrespective of blast count?

a. AML with t(8;21)

Copyright © 2014 Delhi Academy of Medical Sciences, All Rights Reserved. 63/110
b. AML with t(15;17)

c. AML with inv(16)

d. AML with t(6;9)

e. AML with 11q rearrangements

Solution. ABC
AML with t(8;21)(q22;q22.1 ), AML with inv(16)(p13.1q22) or t(16;16)(p13.1 ;q22), and acute
promyelocytic leukaemia with PML-RARA are considered to be acute leukaemias without regard to
blast cell count.

Correct Answer. a,b,c

(150). Which of the following are features of ulcerative colitis

a. Mucosa and submucosa involvement

b. Rectal involvement characteristic

c. Sinuses and fistulas are seen

d. Non caseating granulomas are present

e. Crypt abscesses and cryptitis

Solution. ABE

Copyright © 2014 Delhi Academy of Medical Sciences, All Rights Reserved. 64/110
Correct Answer. a,b,e

(151). Which of the following is/are identified only on supravital stains?

a. Heinz bodies

b. Howell Jolly bodies

c. Basophilic stippling

d. HbH inclusions

e. Pappenheimer bodies

Solution. AD
B, C and E can be seen on routine Romanowsky staining methods

Correct Answer. a,d

(152). All of the following can be used to differentiate between anemia of inflammation and iron deficiency anemia except

a. Serum iron levels

b. Total iron binding capacity

c. Transferrin saturation

d. Hepcidin

e. Zinc protoporphyrin

Solution. ACE

Correct Answer. a,c,e

(153). Not seen in Apert syndrome?

a. Maxillary hypoplasia

b. Syndactyly

c. Mongoloid slant of eyes

d. Craniosynostosis

e. Macrocephaly

Copyright © 2014 Delhi Academy of Medical Sciences, All Rights Reserved. 65/110
Solution. CE
Explanation: Clinical features of Apert Syndrome
o Craniosynostosis: Premature fusion of multiple sutures, including the coronal,
sagittal, squamosal, and lambdoid sutures
o Face: Asymmetric, maxillary hypoplasia, low-set ears
o Eyes: Mild proptosis, hypertelorism with antimongoloid slant of eyes
o Oral cavity: High-arched palate, dental malocclusion, cleft palate
o Hands and feet: Syndactyly of 2nd, 3rd and 4th fingers (Mitten hand)
o Progressive calcification of hands, feet and cervical spine

Correct Answer. c,e

(154). Not seen in a neonate with esophageal atresia is/are?

a. Coughing

b. Abdominal distension

c. Bilious vomiting

d. Respiratory distress

e. Frothy oral secretions

Solution. C
Explanation: The neonate with EA typically has frothing and bubbling at the mouthand nose after
birth as well as episodes of coughing, cyanosis, andrespiratory distress. Feeding exacerbates these
symptoms, causesregurgitation, and can precipitate aspiration. Aspiration of gastric contents
via a distal fistula causes more damaging pneumonitis than aspirationof pharyngeal secretions from
the blind upper pouch. The infantwith an isolated TEF in the absence of EA (“H-type” fistula) might
come to medical attention later in life with chronic respiratory problems,including refractory
bronchospasm and recurrent pneumonias

Correct Answer. c

(155). Which among the following milestone(s) is/are present in a 12-month old child?

a. Object permanence

b. Separation anxiety

c. Radial palmar grasp

d. Self-feeding with a spoon

e. Crawling on flat surface

Solution. ACE
Explanation: Normal ages of attainment of the milestones given above:
• Object permanence: 9 months
• Separation anxiety: 18 months
• Radial palmar grasp: 7 months
• Self-feeding with a spoon: 15 months
Crawling on flat surface: 9 months

Correct Answer. a,c,e

(156). Which among the following is/are a low-risk criteria for fever in a 2 month old child according to Pittsburgh guidelines?

a. Toxic appearance

b. Absence of bacteria on gram stain on urine sample

c. Absence of infiltrates on chest X-ray

d. Dysentery with 10 WBCs/HPF

e. Absolute band cells <1500/μL

Copyright © 2014 Delhi Academy of Medical Sciences, All Rights Reserved. 66/110
Solution. BCE
Explanation: Low-risk criteria for fever in a child aged 1-3 months

Correct Answer. b,c,e

(157). Identify the true statement(s) about cephalhematoma:

a. Localised subperiosteal collection of blood

b. Soft tissue swelling of the scalp

c. Common in forceps injury to periosteum

d. Extends across suture lines

e. Maximum size at birth and then regresses

Copyright © 2014 Delhi Academy of Medical Sciences, All Rights Reserved. 67/110
Solution. A, C
Explanation: Scalp injuries after birth:

Correct Answer. a,c

(158). True statement(s) regarding Bartter syndrome is/are?

a. Hypokalemia

b. Hypercalciuria

c. Metabolic acidosis

d. Azotemia

e. Absence of facial dysmorphism

Copyright © 2014 Delhi Academy of Medical Sciences, All Rights Reserved. 68/110
Solution. AB
Explanation: Bartter syndrome
• Bartter syndrome is a group of disorders characterized by hypokalemic metabolic alkalosis
with hypercalciuria and salt wasting
• Antenatal Bartter syndrome (types I, II, and IV; also called hyperprostaglandin E syndrome)
typically manifests in infancy and has a more-severe phenotype than classic Bartter
syndrome (type III)
• Dysmorphic features, including triangular facies, protruding ears, large eyes with strabismus,
and drooping mouth may be present on physical examination
• Blood pressure is usually normal, although patients with the antenatal form can have severe
salt wasting
• The biochemical features of Bartter syndrome include hypokalemic metabolic alkalosis with
hypercalciuria
• Renal function is typically normal.
• Urinary calcium levels are typically elevated, as are urinary potassium and sodium levels.
• Serum renin, aldosterone, and prostaglandin E levels are often markedly elevated,
particularly in the more-severe antenatal form.
• Nephrocalcinosis, resulting from hypercalciuria, may be seen on ultrasound exam
• Gitelman syndrome is similar but usually presents later, has hypocalciuria and
hypomagnesemia and is virtually devoid of significant facial dysmorphism.

Correct Answer. a,b

(159). Not correct regarding breast feeding jaundice is/are?

a. Due to increased enterohepatic circulation

b. Can produce pathological increase in serum bilirubin

c. High serum bilirubin in 2nd to 4th week of life

d. Can produce kernicterus

e. Breastfeeding need to be stopped

Solution. AE
Explanation: Jaundice in exclusively breast-fed babies
o Breast milk jaundice:
• Significant elevation in unconjugated bilirubin begins to be seen in some breastfed babies
after 1st week. The serum bilirubin rapidly rises and may even reach values between 10-30
mg/dl during the 2nd- 4th week
• Exact cause is unknown. Common theories include presence of glucuronidase in breast milk
or decrease in conjugating ability due to inhibitory substances in breast milk
• Kernicterus may occur as a complication
• Managed conservatively. Phototherapy is effective.
• Some cases may require temporary stoppage of breastfeeding for few days; which rapidly
decreases the serum bilirubin levels (Not a preferred therapy these days)
o Breast feeding jaundice
• Mild elevation in serum bilirubin in the first week of life, in exclusively breast-fed babies.
• Serum bilirubin is usually >12 but rarely exceeds 15 to 16 mg/dL.
• Usually due to decreased milk intake producing dehydration and increased enterohepatic
circulation may also have a role
• Frequent breastfeeding, lactational support and discouraging water or dextrose
supplementation helps in this condition

Correct Answer. a,e

(160). Not a feature of Kwashiorkar?

a. Apathy

b. Desquamation of skin

c. Flaky paint dermatitis

d. Flaky paint dermatitis

e. Relationship with abrupt weaning

Solution. D
Explanation: Baggy pants appearance is seen in Marasmus, not in Kwashiorkar. Other options are
correct.

Correct Answer. d

Copyright © 2014 Delhi Academy of Medical Sciences, All Rights Reserved. 69/110
(161). Not seen in a child with acute ITP?

a. Hepatosplenomegaly

b. Bone pain

c. CNS hemorrhage

d. Epistaxis

e. Lymphadenopathy

Solution. ABE
Explanation: As per Nelson 20th Ed, “the presence of abnormal findings in acute ITP, such as
hepatosplenomegaly, bone or joint pain, remarkable lymphadenopathy other cytopenias, or
congenital anomalies suggests other diagnoses (leukemia, syndromes)”.

Correct Answer. a,b,e

(162). True regarding findings in atrial septal defect (ASD) is/are?

a. Wide fixed split S2

b. No shunt murmur

c. Soft systolic murmur

d. No risk of IE

e. Early diastolic shunt murmur along upper sternal border

Solution. ABCD
Explanation: Physical findings in ostium secondum ASD
• S1 is normal
• In most patients with an ASD, the characteristic finding is that the 2nd heart sound is widely
split and fixed in its splitting during all phases of respiration.
• Shunt murmur is absent
• A systolic ejection murmur is heard; it is medium pitched, without harsh qualities, seldom
accompanied by a thrill, and best heard at the left middle and upper sternal border. It is
produced by the increased flow across the right ventricular outflow tract into the pulmonary
artery, not by low-pressure flow across the ASD.
• A short, rumbling mid-diastolic murmur produced by the increased volume of blood flow
across the tricuspid valve is often audible at the lower left sternal border. This finding, which
may be subtle and is heard best with the bell of the stethoscope, usually indicates a Qp : Qs
ratio of at least 2 : 1.
• Sometimes a pulmonic ejection click can also be heard

Correct Answer. a,b,c,d

(163). Side effect(s) of vigabatrin include(s)?

a. Drowsiness

b. Weight loss

c. Visual field defects

d. Intramyelinic edema

e. Hepatotoxicity

Solution. ACD
Explanation: Vigabatrin is useful in the treatment of partial seizures and infantilespasms.Typical
toxicities include drowsiness, dizziness, and weightgain. Less common but more troublesome
adverse reactions areagitation, confusion, and psychosis. A reversibleintramyelinic edema may be
seen in infants. In addition, long-term therapy with vigabatrin has been associatedwith development
of peripheral visual field defects in 30–50% ofpatients. The lesions are located in the retina, increase
with drugexposure, and are usually not reversible

Correct Answer. a,c,d

Copyright © 2014 Delhi Academy of Medical Sciences, All Rights Reserved. 70/110
(164). Anticancer antibiotics are?

a. Bleomycin

b. Spiramycin

c. Mitomycin

d. Actinomycin D

e. Minocycline

Solution. ACD
Explanation: Anti-tumour antibiotics include
• Actinomycin D
• Bleomycin
• Doxorubicin
• Daunorubicin
• Mitomycin C
• Valrubicin
• Epirubicin
• Idrarubicin

Correct Answer. a,c,d

(165). Uses of alpha-2 agonists include?

a. Relief of spasticity/muscular spasm

b. Erectile dysfunction

c. Terminal cancer pain

d. Conscious sedation in intubated ICU patients

e. Post herpetic neuropathy

Solution. ACD
Explanation: Alpha-2 agonists include Clonidine, methyldopa, guanfacine, guanabenz,
dexmedetomidine (centrally acting), tizanidine (centrally acting)
Clonidine is used in hypertension, diabetic neuropathy, for cancer pain (via epidural or intrathecal
catheters) and in opioid/alcohol withdrawal. Dexmedetomidine is used for sedating conscious
intubated patients in ICU. Tizanidine is used to decrease spasticity and relieve muscular spasms

Correct Answer. a,c,d

(166). Beta-blockers with partial agonist activity include(s)?

a. Carvedilol

b. Carteolol

c. Propranolol

d. Labetalol

e. Pindolol

Copyright © 2014 Delhi Academy of Medical Sciences, All Rights Reserved. 71/110
Solution. BDE

Correct Answer. b,d,e

(167). Drug(s) precipitating serotonin syndrome is/are?

a. Diazepam

b. Linezolid

c. Ondansetron

d. SSRIs

e. Cyproheptadine

Solution. BCD
Explanation: Serotonin syndrome summarised

Copyright © 2014 Delhi Academy of Medical Sciences, All Rights Reserved. 72/110
Correct Answer. b,c,d

(168). Newer anti tubercular drugs is/are?

a. Bedaquiline

b. Delamanid

c. Pretomanid

d. Ceftaroline

e. Etanercept

Solution. ABC
Explanation: New anti-tuberculosis drugs
• Bedaquiline: A diarylquinoline antibiotic that is bactericidal, that interferes with energy
metabolism and replication of tuberculous bacteria. Very effective in multi-drug resistant TB (MDR-TB)
• Delamanid: An orally active anti tuberculous drug, which is a prodrug and forms active
metabolites in body which act by inhibiting the synthesis of mycobacterial cell wall
components, methoxy mycolic acid and ketomycolic acid. Included by WHO in essential
medicine list in 2015-16 for use against MDR-TB.
• Pretomanid: A drug similar to Delamanid, which is active against both replicating and nonreplicating
TB bacilli. Currently in Phase III trials and has shown promising results.
th
Remember: Ceftaroline is a 5 generation cephalosporin, highly active against MRSA. Etanercept is a TNF-α inhibitor, used in various
inflammatory and autoimmune disorders like RA, psoriasis, spondyloarthropathies etc.

Correct Answer. a,b,c

(169). True regarding the antiviral drug trifluridine is/are?

a. Given topically

b. Given systemically

c. Inhibits DNA synthesis

d. Not effective in CMV

e. Useful in acyclovir-resistant HSV

Solution. ACE
Explanation: Trifluridine (trifluorothymidine) is a fluorinated pyrimidinenucleoside that inhibits viral
DNA synthesis in HSV-1, HSV-2,CMV, vaccinia, and some adenoviruses. It is phosphorylated
intracellularlyby host cell enzymes, and then competes with thymidinetriphosphate for
incorporation by the viral DNA polymerase. Incorporation of trifluridine triphosphate intoboth viral
and host DNA prevents its systemic use. Application ofa 1% solution is effective in treating
keratoconjunctivitis andrecurrent epithelial keratitis due to HSV-1 or HSV-2. Cutaneousapplication of
trifluridine solution, alone or in combination withinterferon alfa, has been used successfully in the
treatment ofacyclovir-resistant HSV infections.

Correct Answer. a,c,e

(170). Ivermectin is useful in the management of which among the following parasitic infections?

a. Onchocerciasis

b. Stronglyoides

c. Scabies

d. Schistosomiasis

e. Teniasolium

Solution. ABC
Explanation: Ivermectin is useful in the management of:
i. Onchocerciasis: Treatment is with a single oral dose of ivermectin, 150 mcg/kg,with water
on an empty stomach.
ii. Strongyloidiasis: Treatment consists of two daily doses of 200 mcg/kg
iii. Other Parasites: Ivermectin reduces microfilariae in B malayiand M ozzardiinfectionsbut not
in M perstans infections. It has been used with diethylcarbamazineand albendazole for the
control of W bancroftibutit does not kill adult worms. Is also given orally in scabies patients.

Copyright © 2014 Delhi Academy of Medical Sciences, All Rights Reserved. 73/110
Correct Answer. a,b,c

(171). Effect(s) of digoxin on heart include(s)?

a. Bradycardia

b. Decreased refractory period of AV node

c. Prolonged QT interval

d. Short RR interval

e. Tachycardia at high doses

Solution. AE
Explanation:

Correct Answer. a,e

(172). Drug(s) useful in aspirin-induced asthma include?

a. Montelukast

b. Theophylline

c. Magnesium sulphate

d. Omalizumab

e. Nedocromil

Solution. A
Explanation: Trials with leukotriene inhibitors (like montelukast and zafirlukast) have demonstrated
animportant role for leukotrienes in aspirin-induced asthma. It haslong been known that 5–10% of
asthmatics are exquisitely sensitiveto aspirin, so that ingestion of even a very small dose
causesprofound bronchoconstriction and symptoms of systemic releaseof histamine, such as
flushing and abdominal cramping. Becausethis reaction to aspirin is not associated with any
evidence ofallergic sensitization to aspirin or its metabolites and because it isproduced by any of the
nonsteroidal anti-inflammatory agents, itis thought to result from inhibition of prostaglandin
synthetase(cyclooxygenase), shifting arachidonic acid metabolism from theprostaglandin to the
leukotriene pathway. Support for this ideawas provided by the demonstration that leukotriene
pathwayinhibitors impressively reduce the response to aspirin challengeand improve overall control of asthma on a day-to-day basis.

Correct Answer. a

(173). Most common side effect of phenytoin is:

a. Megaloblastic anemia

b. Gum hypertrophy

c. Hirsutism

d. Peripheral neuropathy

e. Osteomalacia

Solution. B
Explanation: All the options are seen in Phenytoin toxicity, however, Gum hypertrophy is MC,
approx. 20% patients develop this.

Correct Answer. b

Copyright © 2014 Delhi Academy of Medical Sciences, All Rights Reserved. 74/110
(174). Treatment of nocardia infection includes?

a. Ampicillin

b. Fluoroquinolones

c. Azithromycin

d. Cotrimoxazole

e. Amikacin

Solution. ABDE
Explanation: Azithromycin is not effective in Nocardiosis. All others can be used

Correct Answer. a,b,d,e

(175). Pharmacological indication(s) of verapamil include(s)?

a. PSVT

b. Restoring sinus rhythm in Atrial flutter

c. Restoring sinus rhythm in Atrial fibrillation

d. Controlling atrial rate in Atrial flutter

e. Sustained VT

Solution. AD
Explanation: Supraventricular tachycardia is the major arrhythmia indicationfor verapamil.
Adenosine or verapamil are preferred over oldertreatments (propranolol, digoxin, edrophonium,
and vasoconstrictoragents) and cardioversion for termination. Verapamil canalso reduce the
ventricular rate in atrial fibrillation and flutter. Itonly rarely converts atrial flutter and fibrillation to
sinus rhythm.Verapamil is occasionally useful in ventricular arrhythmias.However, intravenous
verapamil in a patient with sustained ventriculartachycardia can cause hemodynamic collapse.

Correct Answer. a,d

(176). Plasma levels of lithium can be increased by which of the following?

a. Valproate

b. Metronidazole

c. Furosemide

d. Tetracyclines

e. Alcohol

Solution. BCD
Explanation: Metronidazole, furosemide and tetracycline cause lithium retention, predisposing to its toxicity

Correct Answer. b,c,d

(177). Antifungals acting by disrupting ergosterol biosynthesis include?

a. 5-Fluorocytosine

b. Caspofungin

c. Amphotericin B

d. Ketoconazole

e. Terbinafine

Copyright © 2014 Delhi Academy of Medical Sciences, All Rights Reserved. 75/110
Solution. DE
Explanation: Site of action of antifungals

Correct Answer. d,e

(178). True about neutrophils:

a. Engulf bacteria

b. Multilobed

c. Granules are slightly basic and stain weakly with azurophilic component of Romanowsky stains

d. Neutrophilia occurs in acute bacterial infections

e. Primary granules also called as specific granules

Solution. ABCD
Explanation: Specific granules are secondary, not primary. Others are correct options.

Correct Answer. a,b,c,d

(179). At physiological pH buffering action in blood is caused by

a. Histidine

b. Methionine

c. Homocysteine

d. Arginine

e. Glycine

Solution. A
Explanation: Major buffers in blood are Plasma proteins, imidazole group of histidine residues in Hemoglobin and carbonic acid
bicarbonate system.

Correct Answer. a

(180). False regarding cerebral blood flow are all except:

a. Normal CBF is 50 ml/100g/min

Copyright © 2014 Delhi Academy of Medical Sciences, All Rights Reserved. 76/110
b. Neuronal death is CBF falls below 5 ml/100 g/min

c. Autoregulation is shown by CBF

d. Autoregulation is impaired in head injury patients

e. CBF depends upon MAP

Solution. ABCDE
Explanation: An example of question having false & except in the statement. The ques is asking
about true statements.
• Normal cerebral blood flow is 50 ml/100 g of brain tissue/minute
• Cell death seen if cerebral blood flow falls below 5 ml/100 g/ min
• Autoregulation is shown by CBF between 50 and 160 mm Hg mean pressure
• CBF depends upon MAP
• This autoregulation gets impaired in head injury patients

Correct Answer. a,b,c,d,e

(181). Crude rod vision under dark conditions is mediated by which among the following ganglion cells?

a. W cells

b. X cells

c. Y cells

d. Z cells

e. All of the above

Solution. A
Explanation: The W cells, constituting about 40 percent of all the ganglion cells, are small, having a
diameter less than 10 micrometers, and they transmit signals in their optic nerve fibers at the slow
velocity of only 8m/sec. These ganglion cells receive most of their excitation from rods, transmitted
by way of small bipolar cells and amacrine cells. They have broad fields in the peripheral retina
because the dendrites of the ganglion cells spread widely in the inner plexiform layer, receiving
signals from broad areas.On the basis of histology, as well as physiological experiments, the W cells
seem to be especially sensitive for detecting directional movement in the field of vision, and they are
probably important for much of our crude rod vision under dark conditions.

Correct Answer. a

(182). Not true regarding GIT hormones is/are?

a. Gastrin may be released in duodenum and jejunum

b. GIP stimulates insulin release

c. Motilin stimulates gastric motility

d. Motilin stimulates intestinal motility

e. Secretin stimulates exocrine pancreas

Copyright © 2014 Delhi Academy of Medical Sciences, All Rights Reserved. 77/110
Solution. ABCDE
Explanation: Table from Guyton 12th Ed

Correct Answer. a,b,c,d,e

(183). Hormone(s) using cAMP as 2nd messenger system include(s)?

a. Calcitonin

b. NO

c. Secretin

d. Insulin

e. Glucagon

Copyright © 2014 Delhi Academy of Medical Sciences, All Rights Reserved. 78/110
Solution. ACE
Explanation:

Correct Answer. a,c,e

(184). Sympathetic stimulation of the heart may include all of the following except?

a. Increased force of contraction

b. Decreased net sodium-calcium permeability

c. Norepinephrine release

d. Increased conduction rates

e. Decreased conduction time from atria to ventricles

Solution. B
Explanation: Mechanism of the Sympathetic Effect on the heart
• Stimulation of the sympathetic nerves releases the hormone norepinephrine at the sympathetic nerve endings. Norepinephrine in turn
stimulates beta-1 adrenergic receptors, which mediate the effects on heart rate. The precise mechanism by which beta-1 adrenergic
stimulation acts on cardiac muscle fibers is somewhat unclear, but the belief is that it increases the permeability of the fiber membrane to
sodium and calcium ions.
• In the sinus node, an increase of sodium-calcium permeability causes a more positive resting potential and also causes increased rate of
upward drift of the diastolic membrane potential toward the threshold level for self-excitation, thus accelerating self-excitation and,
therefore, increasing the heart rate. In the A-V node and A-V bundles, increased sodiumcalcium permeability makes it easier for the action
potential to excite each succeeding portion of the conducting fiber bundles, thereby decreasing the conduction time
from the atria to the ventricles.
-The increase in permeability to calcium ions is at least partially responsible for the increase in contractile strength of the cardiac muscle
under the influence of sympathetic stimulation, because calcium ions play a powerful role in exciting the contractile process of the
myofibrils

Correct Answer. b

(185). True regarding HbA2 is/are?

a. Has more capacity to carry oxygen

b. Concentration is more than HbA

c. Level is increased in thalassemia

d. Consists of 2 alpha and 2 beta chains

e. Levels increased in sickle cell anemia

Copyright © 2014 Delhi Academy of Medical Sciences, All Rights Reserved. 79/110
Solution. C
Explanation: Hemoglobin A2 (HbA2) is a normal variant of hemoglobin A that consists of two alpha and two delta chains (α2δ2) and is found
at low levels in normal human blood. Hemoglobin A2 may be increased in beta thalassemia or in people who are heterozygous for the beta
thalassemia gene. HbA2 exists in small amounts in all adult humans (1.5-3.1% of all hemoglobin molecules) and is
approximately normal in people with sickle-cell disease.

Correct Answer. c

(186). Rupture of ovarian follicles is associated with?

a. Rise in gonadotropins

b. Activation of proteolytic enzymes

c. Fall in gonadotropins

d. hCG surge

e. Rise in prostaglandin levels

Solution. ABE
Explanation: Events during ovulation:
• About 2 days before ovulation, the rate of secretion of LH increases markedly and peaks
about 16 hours before ovulation. FSH secretion also increases at the same time. FSH and LH act synergistically to cause rapid swelling of
the ovarian follicle
• Rate of secretion of estrogen begins to fall about 1 day before ovulation and progesterone secretion begins to rise
• LH surge leads to increased progesterone, which causes release of proteolytic enzymes from theca externa. These enzymes dissolve the
follicular capsular wall.
• Simultaneous formation of new blood vessels and rise in prostaglandins occurs
• This causes plasma transudation into the follicle and its eventual rupture.

Correct Answer. a,b,e

(187). Not considered a neurotrophic factor:

a. IL-3

Copyright © 2014 Delhi Academy of Medical Sciences, All Rights Reserved. 80/110
b. IL-6

c. BDNF

d. IGF-1

e. IGF-2

Solution. A
Explanation: List of neurotrophic factors

Correct Answer. a

(188). True statement(s) regarding sandfly is/are?

a. Size is smaller than mosquitoes

b. Don’t fly long distances

c. Breed in overhead water tanks

d. Endemic in some parts of India

Copyright © 2014 Delhi Academy of Medical Sciences, All Rights Reserved. 81/110
Solution. ABD
• Breed in cracks & crevices
• Bite of female (requires blood meal every 3- 4 days for ovi position)
• Legs twice as long as body
• Flame shaped/ lanceolate wings
• Fine hairs on margins of wings
• Sanitation measures carried out for a distance of 50 feet

Sandfly (contd.)
• Adult flies are nocturnal, spending the day sheltering in dark humid places and in cracks and crevices
• At nightfall they emerge to feed on sugary secretions and plant sap
• Fly is a weak flier and takes short flights to find a victim, adopting a "hopping" style of flight
when close to a host
• The range of flight is 200 yard from their breeding places.
• Adults have short mouthparts and are unable to bite through clothing
• Life Span: Average life of sandfly is about 2 weeks. Comparison between Sandfly and Mosquito

Correct Answer. a,b,d

(189). Which among the following statement(s) are true regarding the vertical national health programmes?

a. Direct finance from centre

b. Skilled manpower

c. Better supervision and monitoring

d. Enhanced community participation

e. Not directly under NRHM

Copyright © 2014 Delhi Academy of Medical Sciences, All Rights Reserved. 82/110
Solution. ABCE
• Vertical Program
• Separate Health Structures with strong central management dedicated to the planning, management & implementation of selected
interventions
• Advantages: Clear objectives & targets motivate staff, operational planning is focused & easy to deliver, efficient & effective delivery,
ability to monitor restricted output
• Disadvantages: No capacity to accommodate extra work in disasters, resources used for specific
activities only, deskilling of health worker, no focus on overall development, dependent on donors
for funding, placement of workers after completion is challenging, long term public motivation not sustained and may not be cost effective
in long run
• Integrated/umbrella health programs: Better
• with better supervision, monitoring and skilled manpower with a flexible community based approach.
• Example National health mission (NHM)

Correct Answer. a,b,c,e

(190). Follow-up bias is seen in which among the following?

a. Prospective studies

b. Retrospective studies

c. Cohort studies

d. Case series

e. Cross sectional studies

Solution. AC
• Follow-up bias (due to differential follow-up or loss to follow-up) is typically a problem of prospective, cohort studies. However, recall
bias is not seen in these study models

Correct Answer. a,c

(191). A test has been introduced that will detect a certain disease 1 year earlier than it is usually detected which of the following is not likely to
happen to the disease with in 10 years after test introduced? (assuming that 1 year early detection has no effect on the natural history of
disease)

a. It will be a good screening test

b. Period prevalence will decrease

c. Apparent 5 year survival rate will decrease

d. Incidence rate will decrease

e. Apparent 5 year survival rate will increase

Solution. A; B; C; D
A. It will be a good screening test: No as this is example of lead time bias
B. Period prevalence will decrease: No it will increase as P = I * D (now duration has increased by
1 year)
C. Apparent 5 year survival rate will decrease: actually it increases mathematically
D. Incidence rate will decrease: Incidence rate over 10 year period will stay constant , as if not this year then it would have been
diagnosed the next year

Correct Answer. a,b,c,d

Copyright © 2014 Delhi Academy of Medical Sciences, All Rights Reserved. 83/110
(192). 6 prospective cohort studies were undertaken to examine the association between anti oxidant consumption & delivery of premature
baby. The results of these 6 hypothetical studies are
illustrated in the following figure & are expressed as RR with 95% CI. Which is the following are correct?

a. Maximum sample size is for study F

b. Minimum sample size is for study E

c. Maximum statistical significance is for study F

d. Minimum statistical significance is for study E

e. Study D is more significant than study F

Solution. ABDE
• If confidence limits touch the null/ neutral value it becomes insignificant
• More the point estimate, better it is
• Narrow CI is better
• Larger sample size
• Smaller SD
• Sequence of Using the rules: A> B> C

Correct Answer. a,b,d,e

(193). The dimension indices comprising Human development index (HDI) include the following except:

a. Quality of life index

b. Child mortality index

c. Life expectancy index

d. Education index

e. GNI Index

Copyright © 2014 Delhi Academy of Medical Sciences, All Rights Reserved. 84/110
Solution. AB

Correct Answer. a,b

(194). Disease(s) spread by tick include?

a. Lyme’s disease

b. Anaplasmosis

c. Epidemic typhus

d. Endemic typhus

e. Tularemia

Copyright © 2014 Delhi Academy of Medical Sciences, All Rights Reserved. 85/110
Solution. ABE
• Disease(s) spread by tick include?
A. Lyme’s disease
B. Anaplasmosis
C. Epidemic typhus: louse
D. Endemic typhus: rat flea
E. Tularemia
Rickettsial Diseases (v.v.imp)

• Also serve as arthropod reservoir, by maintaining the rickettsiae through ovarian transmission
• Bite causes an eschar
• Arthropod insect vectors (except Q Fever)

Copyright © 2014 Delhi Academy of Medical Sciences, All Rights Reserved. 86/110
Correct Answer. a,b,e

(195). True regarding Injectable polio vaccine (IPV) is/are all except?

a. Dose is 0.1 ml

b. Dose is 0. 5ml

c. Given intradermal

d. Given Intramuscular

e. Cannot be given if OPV administered in last 4 weeks

Solution. E
• India started IPV as i.m. single dose 0.5 ml and then we switched to 0.1 ml two doses at 6 and 14 weeks along with OPV

Correct Answer. e

(196). True statement(s) regarding demography census in India is/are?

a. 1921 was the year of great divide

b. Currently in low stationary phase

c. First census done in 1891

d. Dependency ratio is < 40% currently

e. Population pyramid is almost cylindrical in India currently

Solution. A
A. 1921 was the year of great divide: CORRECT
B. Currently in low stationary phase: WRONG; it is late expanding
C. First census done in 1891: WRONG; it is 1881
D. Dependency ratio is < 40% currently: WRONG; it is 52.4%
E. Population pyramid is almost cylindrical in India currently: WRONG; it is globular
Demographic Cycle

Correct Answer. a

(197). Not true regarding autistic spectrum disorders is/are?

Copyright © 2014 Delhi Academy of Medical Sciences, All Rights Reserved. 87/110
a. Non-verbal skills more severely affected than verbal skills

b. Lack of social skills

c. Intellectual dysfunction seen in most cases

d. Behavioural disturbances are common

e. Clinical depression in adolescents

Solution. AC
Explanation: In most epidemiologically based samples of persons with autisticdisorder,
approximately 50% exhibit severe or profound intellectualdisability, 35% exhibit mild to moderate
intellectual disability, and theremaining 20% have IQs in the normal range. Verbal skills are
typicallymore impaired than nonverbal skills. Intellectual impairment is not anessential diagnostic
feature of autism. A range of behavioral difficulties can be observed in ASD includinghyperactivity,
obsessive compulsive phenomena, self-injury, aggression,stereotypies, tics, and affective symptoms,
Overt clinical depressionis sometimes observed, and this may be particularly true foradolescents.

Correct Answer. a,c

(198). Abnormalities in brain seen in a patient with schizophrenia may include?

a. Reduced size of the ventricles

b. Reduced cortical gray matter

c. Reduced neuropil

d. Increased GAD-1 expression

e. Reduced pre-frontal cortical activity

Solution. BCE
Explanation: In schizophrenia, structural and functional imaging studies have confirmed earlier
pathologic studies that show enlargement of the ventricular system and reduction of cortical and
subcortical gray matter in frontal and temporal lobes and in the limbic system. Functional imaging
studies show reduced metabolic (presumably neural) activity in the dorsolateral prefrontal cortex at
rest and when performing tests of executive function, including working memory. There is also
evidence for impaired structural and task-related functional connectivity, mainly in frontal and
temporal lobes. The reduction in cortical thickness seen in schizophrenia is associated with increased
cell packing density and reduced neuropil (defined as axons, dendrites, and glial cell processes)
without an apparent change in neuronal cell number. Specific classes of interneurons in prefrontal
cortex consistently show reduced expression of the gene encoding the enzyme glutamic acid
decarboxylase 1 (GAD1), which synthesizes γ-aminobutyric acid (GABA), the principal inhibitory
neurotransmitter in the brain.

Correct Answer. b,c,e

(199). Which among the following statement(s) is/are true regarding Duloxetine?

a. Belongs to TCA class of drugs

b. Can be used in major depressive disorder

c. Useful in generalised anxiety

d. Contraindicated in diabetic neuropathy

e. Caution needed in hepatic disease

Solution. BCE
Explanation: Duloxetine has been approved for the treatment of depression and neuropathic pain in
August 2004. Duloxetine is contraindicated in patients with heavy alcohol use or chronic liver
disease, as duloxetine can increase the levels of certain liver enzymes that can lead to
acute hepatitis or other diseases in certain at risk patients. Currently, the risk of liver damage
appears to be only for patients already at risk, unlike the antidepressant nefazodone, which, though
rare, can spontaneously cause liver failure in healthy patients. Duloxetine is also approved for major
depressive disorder (MDD), generalized anxiety disorder (GAD), diabetic neuropathy, chronic
musculoskeletal pain, including chronic osteoarthritis pain and chronic low back pain.

Correct Answer. b,c,e

Copyright © 2014 Delhi Academy of Medical Sciences, All Rights Reserved. 88/110
(200). Anti-craving agents for alcohol withdrawal include?

a. Naltrexone

b. Disulfiram

c. Metronidazole

d. Fluoxetine

e. Acamprosate

Solution. ADE
Explanation: Anti-craving agents for alcohol wiithdrawl:
- Acamprosate
- Naltrexone
- Fluoxetine
- Topiramate
- Onadansetron


Correct Answer. a,d,e

(201). Management of phobias includes?

a. Propranolol

b. Venlafaxine

c. Sertraline

d. Chlorpromazine

e. Behavior therapy

Solution. ABCE
Explanation: Beta blockers (e.g., propranolol, 20–40 mg orally 2 h before the event) are particularly
effective in the treatment of “performance anxiety” (but not general social phobia) and appear to
work by blocking the peripheral manifestations of anxiety such as perspiration, tachycardia,
palpitations, and tremor. MAOIs alleviate social phobia independently of their antidepressant
activity, and paroxetine, sertraline, and venlafaxine have received FDA approval for treatment of
social anxiety. Benzodiazepines can be helpful inreducing fearful avoidance, but the chronic nature
of phobic disorders limits their usefulness. Behaviorally focused psychotherapy is an important
component of treatment because relapse rates are high when medication is used as the sole
treatment.

Correct Answer. a,b,c,e

(202). Defence mechanisms in OCD include

a. Consolidation

b. Isolation of affect

c. Undoing

d. Reaction formation

e. Displacement

Solution. BCDE
Explanation: Defence mechanisms in OCD include: Isolation of affect, undoing, reaction formation,
displacement.

Correct Answer. b,c,d,e

(203). Chest X-ray views performed in suspected pleural effusion may include?

a. Supine PA

b. Standing PA

c. Supine AP

Copyright © 2014 Delhi Academy of Medical Sciences, All Rights Reserved. 89/110
d. Oblique

e. Decubitus

Solution. BCE
Explanation: Radiographic examination in pleural effusion shows a generally homogeneous
densityobliterating the normal markings of the underlying lung. Small effusionsmay cause
obliteration of only the costophrenic or cardiophrenicangles or a widening of the interlobar septa.
Examinations should beperformed with the patient both supine and upright, to demonstrate ashift
of the effusion with a change in position; the decubitus positionmay be helpful.

Correct Answer. b,c,e

(204). Which of the following is TRUE regarding MRI?

a. MRI is better than CT scan for bony lesion

b. Grey matter is grey on T1W1 images

c. Uses gadolinium as the dye

d. Gadolinium is safer than iodine based contrast agents

e. Can be used in multiple planes

Solution. BCDE
Explanation: MRI can be done in multiple planes and is better than CT scan for soft tissues. CT,
however, is better for Calcified tissues and bony structures. MRI uses gadolinium contrast, which is
safer than Iodinated contrast materials. White matter is darker than grey matter in T1 and brighter
in T2 images.

Correct Answer. b,c,d,e

(205). Helical CT scan abdomen has the following differences from non-helical CT scan?

a. Increased respiratory misregistration in helical CT

b. Allows imaging in both the arterial and the portal venous phase with a single contrast injection

c. Continuous data acquisition

d. Lower iodine dose needed for hepatic enhancement

e. Is a suitable replacement for IVU & angiography in the assessment of renal donors.

Solution. BCDE
Explanation: Spiral CT scanners have a number of potential advantages over conventional scanners,
including the absence of respiratory misregistration, overlapping slices produced, optimization of IV
contrast enhancement & use even as a replacement for invasive procedures like angiography.

Correct Answer. b,c,d,e

(206). Correct regarding Gray (Gy) in radiotherapy?

a. SI unit of radiation absorbed

b. 1 Gy= 100 J/kg

c. 1 Gy=100 cGy

d. 1 Gy = 1 rad

e. 1 cGy= 1 rad

Solution. ACE
Explanation: The International System (SI) unit for radiation absorbed is the Gray (Gy): 1 Gy refers to
1 J/kg of tissue; 1 Gy equals 100 centigrays (cGy) of absorbed dose. A historically used unit appearing
in the oncology literature, the rad (radiation absorbed dose), is defined as 100 ergs of energy
absorbed per gram of tissue and is equivalent to 1 cGy

Correct Answer. a,c,e

Copyright © 2014 Delhi Academy of Medical Sciences, All Rights Reserved. 90/110
(207). Which among the following conditions are associated with pulmonary oligemia on radiography?

a. TGA

b. ASD

c. Truncus arteriosus

d. TOF

e. Ebstein’s Anomaly

Solution. DE
Explanation: Conditions associated with pulmonary oligemia on CXR includes
• TOF
• Pulmonary atresia
• Tricuspid atresia
• Ebstein’s anomaly
• Eisenmenger syndrome

Correct Answer. d,e

(208). Which among the following are not the radiological changes seen in Mitral stenosis?

a. Straightened left border of heart

b. LV hypertrophy

c. Decreased intercarinal angle

d. Bulge in anterior aspect of aorta

e. Double atrial shadow

Solution. BCD
Explanation: Radiological changes seen in Mitral stenosis:
• Double atrial shadow
• Straightened left border of heart
• Increased carinal angle; leading to splaying of mainstem bronchus
• Small aorta, small sized LV
• RV hypertrophy
• Prominence of upper lobe vasculature(Antler sign)
• Kerley lines due to interstitial edema
• Bat-wing appearance
• Brown induration (due to deposition of hemosiderin-laden macrophages)

Correct Answer. b,c,d

(209). Appearance of ADPKD on imaging includes:

a. Spider leg appearance on urography

b. Enlarged kidney

c. Decreased size of kidney

d. 3 or more cysts seen on USG

e. Bell appearance on urography

Solution. ABDE
Explanation: Imaging features of Autosomal dominant polycystic kidney disease (ADPKD) include the
following:
• X-ray: Multiple calcifications in ring distribution
• Ultrasound (Screening test of choice): Shows enlarged kidney, multiple bilateral cysts.
Presence of 3 or more cysts(unilaterally or bilaterally) has been suggested as a criteria for
the diagnosis of ADPKD in people aged 15-40 years.
• IVP: Shows spider leg appearance
• CT scan: Shows multiple, well-defined, thin-walled cysts
• MRI: T1 shows low signal, T2 shows high signal; multiple cysts visualised

Correct Answer. a,b,d,e

Copyright © 2014 Delhi Academy of Medical Sciences, All Rights Reserved. 91/110
(210). Air crescent sign on CXR seen in

a. Aspergilloma

b. Inspissated pus in a cavity

c. Tumor or clot within cavity

d. Hydatid cyst

e. Pneumocystis carinii

Solution. ABCD
Explanation: Pneumocystis shows perihilar opacities on CXR.

Correct Answer. a,b,c,d

(211). Not true regarding extradural hematoma?

a. Damage to bridging veins

b. Lucid interval seen

c. Biconvex hyperdensity on CT scan

d. Immediate surgery is needed

e. Mortality upto 50%

Solution. BCD
Explanation: Extradural hematoma
• Extradural haematoma is a neurosurgical emergency. It results from rupture of an artery,
vein or venous sinus, in association with a skull fracture.
• Typically, it is damage to the middle meningeal artery under the thin temporal bone.
• A low energy injury mechanism, perhaps with brief loss of consciousness, is sufficient to
start the extradural bleeding. The patient may then present in the subsequent lucid interval
with headache, but without any neurological deficit. Later, rapid deterioration follows. There
is contralateral hemiparesis, reduced conscious level and ipsilateral pupillary dilatation, the
cardinal signs of brain compression and herniation.
• On CT, extradural haematomas appear as a lentiform (lens shaped or biconvex) hyperdense
lesion between skull and brain, constrained by the adherence of the dura to the skull.
• Extradural haematoma requires immediate transfer to the most accessible neurosurgical
facility, for immediate evacuation in deteriorating or comatose patients or those with large
bleeds, and for close observation with serial imaging in all cases.
• Overall mortality is around 10–20 per cent, but is considerably lower in isolated extradural
haematoma.

Correct Answer. b,c,d

(212). Which among the following is/are non-absorbable suture(s)?

a. Chromic catgut

b. Polygluconate

c. Polypropylene

d. Polyglactin

e. Silk

Copyright © 2014 Delhi Academy of Medical Sciences, All Rights Reserved. 92/110
Solution. CE

Correct Answer. c,e

(213). True regarding post-cholecystectomy bile duct injury is/are?

a. More common in laparoscopic and less in open surgery

b. Cholangiography useful in identification

c. Biliary peritonitis is common

d. Simple ligation is not recommended in even minor leaks

e. Injuries adjacent to bifurcation should not be reanastomosed

Copyright © 2014 Delhi Academy of Medical Sciences, All Rights Reserved. 93/110
Solution. BC
Explanation: Iatrogenic bile duct injury
Iatrogenic bile duct injury: More than 80% of all iatrogenic bile duct injuries occur during
cholecystectomy, and they can occur in the open or laparoscopic setting. Inflammation in the porta,
variable biliary anatomy, inappropriate exposure, aggressive attempts at hemostasis, and
inexperience of the surgeon are commonly cited risk factors.
Factors associated: Although early reports suggested that surgical inexperience (performing fewer
than 20 laparoscopic cholecystectomies) was highly correlated with bile duct injury, evidence has
suggested that visual misperception accounts for 97% of iatrogenic biliary injuries and technical skill
or knowledge accounts for only 3%. Although the use of routine versus selective cholangiography is
controversial, evidence has suggested that cholangiography does not completely avoid bile duct
injury but may reduce the incidence and extent of injury and allow immediate recognition and
management.
Presentation: Bile duct injury may be identified intraoperatively but usually is manifested in the
postoperative period. Leak of bile into the peritoneal cavity, with subsequent bile peritonitis, tends
to be manifested earlier than bile duct stricture and its associated jaundice. In the setting of bile
leakage, patients may present with fever, increasing abdominal pain, jaundice, or bile leakage from
an incision. Alternatively, injury to the bile duct that does not leak bile will usually be manifested
with jaundice, with or without pain. Overall, only 10% of postoperative bile duct strictures are
recognized within the first week, and approximately 70% are diagnosed within 6 months of the
original operation.
Treatment: When bile duct injury is suspected intraoperatively, conversion to an open operation
and use of cholangiography help delineate management. In the adult, for ducts smaller than 3 mm
that by cholangiography drain only a single segment or subsegment of liver, simple ligation should
suffice for management. Ducts larger than 3 mm usually drain more than a single segment of liver
and thus, if transected, should be re-implanted into the biliary tree. If the injury occurs to a larger
duct but is not caused by electrocautery and involves less than 50% of the circumference of the wall,
a T tube placed through the injury, which is effectively a choledochotomy, usually will allow healing
without the need for subsequent biliary-enteric anastomosis. More commonly, injuries occur
adjacent to the bifurcation or involve more than a 1-cm defect between the ends of the bile duct.
These injuries require re-anastomosis to the gastrointestinal tract

Correct Answer. b,c

(214). True about abdominal compartment syndrome is/are?

a. Normal pressure in abdomen is upto 5 mm Hg

b. Intra-abdominal pressure is measured by bladder pressure

c. ARDS and cerebral edema may occur

d. Renal failure is an early sign

e. Abdominal decompression by surgical opening is the therapy of choice

Solution. ABCE
Explanation: Normal intraabdominal pressure is 0 to 5 mm Hg
• Abdominal compartment syndrome (ACS) is a complication of abdominal sepsis, trauma and
conditions like aneurysm rupture.
• High pressure above 25 to 30 mm Hg can lead to tissue ischemia and multiorgan failure like
ARDS, Oliguria, Cerebral edema, cardiac insufficiency and cardiac arrest. These are late signs.
• Bladder pressure can be used to measure the intraabdominal pressure
• Relieving the tension by surgical incision or laparostomy is recommended

Correct Answer. a,b,c,e

(215). True regarding small bowel resection?

a. Bile acid malabsorption can lead to diarrhea

b. If jejunum is cut, ileum can compensate jejunal function

c. If ileum is cut, jejunum can compensate ileal function

d. Ileal resection is better tolerated due to jejunal compensation

e. Ileocecal valve resection does not affect prognosis

Solution. ABC
Explanation: During small bowel resection, jejunum can compensate for removal of ileum and vice
versa. Jejunal resection is better tolerated than ileal resection since ileum can compensate better.
Resection of upto 70% small bowel can be tolerated if the terminal ileum and ileocaecal valve are
preserved. Bile acid malabsorption can lead to diarrhea.

Copyright © 2014 Delhi Academy of Medical Sciences, All Rights Reserved. 94/110
Correct Answer. a,b,c

(216). True statement(s) regarding ballistic injury of low velocity type is/are?

a. Low infection rate

b. Brain and spinal cord injury require bullet fragment removal in case of significant compression

c. No role of prophylactic antibiotics

d. Conservative management indicated in superficial abdominal injury

e. Exploratory laparotomy indicated in abdominal injury

Solution. ABDE
Explanation: Low velocity gunshot wounds
• Have a low infection rate and prophylactic antibiotics started and give for 48 hours
• Decompression not needed, except ion brain and spinal cord compression
• No role of steroids
• Superficial wounds may be conservatively treated
• Deep abdominal wounds need exploratory laparotomy

Correct Answer. a,d,e

(217). Not true regarding Dieulafoy’s disease?

a. AV malformation

b. Occurs in stomach

c. Covered by intact normal mucosa in most patients

d. Excision is avoided

e. Bleeding is common

Solution. D
Explanation: Dieulafoy’s disease
This is essentially a gastric arterial venous malformation thathas a characteristic histological
appearance. Bleeding due tothis malformation is one of the most difficult causes of
uppergastrointestinal bleeding to treat. The lesion itself is covered bynormal mucosa and, when not
bleeding, it may be invisible. Ifit can be seen while bleeding, all that may be visible is
profusebleeding coming from an area of apparently normal mucosa. Ifthis occurs, the cause is
instantly recognisable. If the lesion canbe identified endoscopically there are various means of
dealingwith it, including injection of sclerosant and endoscopic clips. Ifit is identified at operation
then only a local excision is necessary.Occasionally, a lesion is only recognised after gastrectomyand
sometimes not even then. The pathologist, as well as theendoscopist, may have difficulty in finding
it.

Correct Answer. d

(218). Which among the following is/are included in Class 3 surgical wound?

a. Opening of genitourinary system with infected urine

b. Surgical incision over a clean site

c. Rectal wound with faecal contamination

d. Perforated appendix

e. Incision given through an abscess

Copyright © 2014 Delhi Academy of Medical Sciences, All Rights Reserved. 95/110
Solution. C
Explanation: Wound classification

Correct Answer. c

(219). Identify the true statement(s) regarding surgical occlusive dressing:

a. Allows no entry of bacteria from outside

b. Used for exudative wounds

c. Retains moisture

d. Wide mesh is used

e. Provides mechanical support

Solution. A, C, E
Explanation: Wound dressings
- Wound dressings can be divided into various subtypes: Nonadherent fabrics, Absorptive,
Occlusive and miscellaneous.
- Occlusive dressings have the benefit of providing moisture retention, mechanical protection
and act as a barrier for the entry of bacteria.
- Occlusive dressings are preferred in wounds like
- Superficial wounds
- Eschars
- Granulating wounds
- Deep wounds.
- Occlusive dressings are not preferred in exudative wounds. In exudative wounds, absorptive
dressings are preferred. A wide mesh is also a feature of absorptive dressings
- Types of occlusive dressings
a) Non biologic: Films, Alginates, Hydrocolloids, Hydrogels
b) Biologic: Homograft, Xenograft, Amnion, Skin substitutes

Correct Answer. a,c,e

(220). Indication(s) of percutaneous gastrostomy is/are?

a. Aspiration risk

b. Mechanical ventilation needed for more than 2 days

c. Severe facial trauma

d. Dementia with severe malnutrition

e. Severe cachexia

Solution. ACD
Explanation: Accepted primary indications for a percutaneous gastrostomy (PEG) or percutaneous
gastrojejunostomy (PEGJ) include inability to swallow, high risk of aspiration, severe facial trauma,
and indications for mechanical ventilation for longer than 4 weeks. Other indications include
nutritional access for debilitated patients and patients with dementia with severe malnutrition.

Correct Answer. a,c,d

(221). True statement (s) regarding axillary lymph node dissection in breast cancer is/are?

Copyright © 2014 Delhi Academy of Medical Sciences, All Rights Reserved. 96/110
a. Arm kept at 90 degrees during the procedure

b. Can be done through mastectomy incision

c. Only Level I nodes are removed in modified radical mastectomy

d. Halsted’s procedure preserves lateral pectoral nerve and pectoralis minor

e. In Halsted’s radical mastectomy, level I, II and III axillary lymph nodes are removed along with skin, nipple, areola, all breast tissue,
pectoralis major and pectoralis minor

Solution. ABE
Explanation:

Axillary LN dissection
• Removal of all lymph nodes and fatty tissue from the axillary space
• Patient is positioned in supine position, with head end raised by 15 to 30 degrees, with arm abducted and at 90 degrees to the chest
wall.
• Dissection can be carried out through the mastectomy incision or may need a separate incision.

Correct Answer. a,b,e

(222). You are called to the emergency department to evaluate a 55-year-old woman following motor vehicle crash with associated head trauma.
She withdraws to pain and is intubated for airway protection. In order to calculate the Glasgow Coma Scale score, which of the following
components of the neurologic examination are necessary?

a. Motor response, verbal response, corneal reflexes

b. Motor response, eye opening, verbal response

c. Eye opening, pupillary light reflexes, motor response

d. Pupillary light reflexes, motor response, verbal response

e. Corneal reflexes, pupillary light reflexes, motor response

Copyright © 2014 Delhi Academy of Medical Sciences, All Rights Reserved. 97/110
Solution. B
Explanation: Glasgow Coma Scale (GCS)

Correct Answer. b

(223). Not true regarding Gorlin’s syndrome is/are?


th
a. Begins in 4 decade

b. Basal cell carcinomas

c. Jaw cysts

d. Autosomal recessive

e. High mortality in childhood

Solution. ADE
Explanation: Gorlin’s syndrome is genetically determined, a disorder of childhood onset. Along with
multiple basal cell carcinomas, other abnormalities include skin ribs on the palms and soles,
epithelial jaw line cysts, rib abnormalities, ectopic calcifictions in the dura, and mental retardation.
The disease is transmitted as autosomal dominant with no sex linkage. Generally the tumors have a
benign clinical course until after puberty.

Correct Answer. a,d,e

(224). An 85-year-old ventilator-dependent male was endotracheally intubated 10 days ago. He remains unresponsive and is not a candidate for
early extubation. The intensive care unit (ICU)
attending elects to perform tracheotomy at the bedside. During the procedure, copious dark blood is encountered. This is most likely due
to transection of which of the following?

a. Anterior jugular vein

b. External jugular vein

c. Internal jugular vein

d. Middle thyroid vein

e. Inferior thyroid vein

Solution. A
Explanation: The anterior jugular vein can cross the midline overlying the proximal trachea. Midline
cervical dissection without adequate visualization can injure the vein and require open surgical
repair. The other veins do not cross the midline and are not generally at risk in tracheotomy.

Correct Answer. a

Copyright © 2014 Delhi Academy of Medical Sciences, All Rights Reserved. 98/110
(225). Management of choledochal cyst include(s)?

a. Observation

b. Radical excision

c. Biliary tract reconstruction

d. Hepatic resection

e. Deroofing

Solution. BC
Explanation: Radical excision of the cyst is the treatment of choice with reconstruction of the biliary
tract using a Roux-en-Y loop of jejunum. Complete resection of the cyst is important because of the
association with the development of cholangiocarcinoma. Resection and Roux-en-Y reconstruction is
also associated with a reduced incidence of stricture formation and recurrent cholangitis.

Correct Answer. b,c

(226). Anterior triangle of neck masses include(s)?

a. Thyroglossal cyst

b. Carotid body tumour

c. Supraclavicular lymph nodes

d. Branchial cyst

e. Submandibular gland

Solution. BDE
Explanation: Anterior triangle of the neck masses
• Lymph nodes along the anterior border of SCM muscle
• Thyroid nodule
• Submandibular gland
• Branchial cyst and fistula
• Carotid body tumour
• Carotid aneurysm
• Laryngocele
• Pharyngeal pouch

Correct Answer. b,d,e

(227). Treatment of retroperitoneal fibrosis include(s)?

a. Corticosteroids

b. Ureterolysis

c. Ureteral stenting

d. Methysergide therapy

e. Localised radiotherapy

Solution. ABC
Explanation: Treatment of retroperitoneal fibrosis
• Once malignancy, drug-induced, and infectious etiologies are ruled out, treatment of the
retroperitoneal fibrotic process is instituted.
• Corticosteroids, with or without surgery, are the mainstay of medical therapy.
• Surgical treatment consists primarily of ureterolysis or ureteral stenting and is required in
patients who present with significant hydronephrosis.
• Laparoscopic ureterolysis has been shown to be as efficacious as open the open procedure.
• Patients with iliocaval thrombosis require anticoagulation, although appropriate duration of
therapy is uncertain.
• Endovascular interventions for iliocaval occlusion have also been shown to be effective in
small numbers of patients.

Correct Answer. a,b,c

Copyright © 2014 Delhi Academy of Medical Sciences, All Rights Reserved. 99/110
(228). Milan criteria is used in determining which type of transplantation?

a. Liver transplant

b. Kidney transplant

c. Cornea transplant

d. BM transplant

e. Stem cell transplant

Solution. A
Explanation: The Milan criteria are a generally accepted set of criteria used to assess suitability in
patients for liver transplantation with cirrhosis and hepatocellular carcinoma. In order to be suitable
for a liver transplantation, one needs to have:
• Single tumour with diameter ≤5 cm, or up to 3 tumours each with diameter ≤3 cm
• No extra-hepatic involvement
• No major vessel involvement?

Correct Answer. a

(229). Regarding pleomorphic adenoma, which among the following is/are true?

a. Parotid gland is most commonly involved

b. Malignant transformation is not seen

c. Also called mixed tumour

d. More common in men than women

e. Superficial parotidectomy is treatment of choice

Solution. ACE
Explanation: Pleomorphic adenoma (Mixed tumour)
• MC tumour of salivary gland and MC involves parotid gland
• Mostly benign but malignant transformation known to occur, with increasing duration
• Females slightly more commonly affected than males
• Radioresistant, so surgical management by superficial parotidectomy is the therapy of
choice

Correct Answer. a,c,e

(230). False statements regarding bronchogenic cyst are all except:

a. Most commonly seen in anterior mediastinum

b. Presents as wheezing

c. Large cyst can produce pressure symptoms

d. Can be pre-malignant

e. Mostly asymptomatic

Solution. BCDE
Explanation: Bronchogenic cysts
- Most common primary cysts in the mediastinum. They are mostly seen in middle mediastinum,
near the carina
- They are a kind of duplication cysts originating from the airways.
- Mostly asymptomatic, they may sometimes produce pressure effects (e.g. wheeze, recurrent
pneumonias or atelectasis). May also get infected to produce fever and cough
- Histologically, they are hamartomatous and usually consist of a single cyst lined by epithelium.
The mesenchyme may contain cartilage and smooth muscle
- CXR shows a dense mass. CT is diagnostic: Shows a spherical fluid or mucus filled non-enhancing mass
- Some cases can be pre-malignant.
- Treatment is simple or segmental resection by VATS or thoracotomy. Rarely, lobectomy may be needed.

Correct Answer. b,c,d,e

Copyright © 2014 Delhi Academy of Medical Sciences, All Rights Reserved. 100/110
(231). True statements regarding fat necrosis of breast?

a. >90% give history of preceding trauma

b. Non-malignant

c. Biopsy is usually needed

d. More common in lactating women

e. Mammography can differentiate it from carcinoma

Solution. BC
Explanation: Fat necrosis of the breast/traumatic fat necrosis
- May be acute or chronic and seen commonly in middle aged females. No relation with lactation.
- A painless lump is seen following a blow or trauma. History of trauma may even be absent, in as high as 50% patients
- May mimic malignancy due to tethering of skin, nipple retraction and mass. May even get
calcified in some cases
- Non-malignant condition
- Mammography cannot differentiate from malignancy
- Needle biopsy is always needed in doubtful cases.

Correct Answer. b,c

(232). Over the past 6 months, a 60-year-old woman with long standing duodenal ulcer disease has been complaining of anorexia, nausea,
weight loss and repeated vomiting. She recognizes
undigested food in the vomitus. Examination and workup reveal dehydration, hypokalemia, and hypochloremic alkalosis. What is the most
likely diagnosis?

a. Carcinoma of the fundus

b. Penetrating ulcer

c. Pyloric obstruction due to cicatricial stenosis of the lumen of the duodenum

d. ZES (Zollinger Ellison Syndrome)

e. Anorexia nervosa

Solution. C
Explanation: Chronic duodenal ulcer, with recurrent episode of healing and repair, may lead to
pyloric obstruction due to scarring and stenosis of the duodenum. Painless vomiting of undigested
food may occur once or twice a day. Surgical intervention should be carried out after correction of
fluid and electrolyte imbalances. Preoperative antibiotics should be used due to bacterial
overgrowth secondary to gastric stasis.

Correct Answer. c

(233). Complications of diverticulitis include all except?

a. Carcinoma of the colon

b. Extraintestinal manifestations such as arthritis, iritis, and skin rashes

c. Fistulisation to adjacent organs such as the bladder, with colovesical fistula

d. Arteriovenous fistulae of the intestine

e. Sclerosing cholangitis

Solution. ABDE
Explanation: Diverticulitis results from acute inflammation of a colonic diverticula. The process may
extend into adjacent organs (e.g., the urinary bladder and a fistula between the colon and bladder
colo-vesical fistula may ensue). This leads to passage of colonic gas and fecal material into the
bladder and urine resulting in pneumaturitis and fecaluria. Sigmoid resection and repair of the
bladder fistula is indicated.

Correct Answer. a,b,d,e

(234). A 40-year-old alcoholic male is admitted with severe epigastric pain radiating to the back. Serum amylase level is reported as normal, but
serum lipase is elevated. The serum is noted to be milky in appearance. A diagnosis of pancreatitis is made. The serum amylase is normal
because?

Copyright © 2014 Delhi Academy of Medical Sciences, All Rights Reserved. 101/110
a. The patient has chronic renal failure.

b. The patient has hyperlipidemia.

c. The patient has alcoholic cirrhosis

d. The patient has alcoholic hepatitis.

e. The diagnosis of pancreatitis is incorrect

Solution. B
Explanation: In pancreatitis, the serum amylase level may be normal. The causes include:
i. Hyperlipidemia, which interferes with chemical determination of amylase;
ii. Increased urinary excretion of amylase; and
iii. Near complete destruction of pancreatic parenchyma as a result of chronic pancreatitis. On
the other hand, the serum amylase level may be elevated in the absence of pancreatitis (for
example, perforated peptic ulcer, gangrenous cholecystitis, small bowel strangulation or
chronic renal failure.)

Correct Answer. b

(235). Seen in tracheo-esophagealfistua with distal esophageal atresia?

a. VACTERL anomalies are commonly associated

b. Oligohydramnios in mother

c. Drooling of saliva

d. Respiratory distress with abdominal distension

e. Nasogastric tube fails to be passed into the stomach

Copyright © 2014 Delhi Academy of Medical Sciences, All Rights Reserved. 102/110
Solution. ACDE
Explanation:Esophageal atresia (EA) and Tracheo-esophageal fistula (TEF)

- Esophageal atresia (EA) is the most common congenital anomaly of the esophagus, with a
prevalence of 1.7 per 10,000 live births. Of these, >90% have an associated tracheoesophageal
fistula (TEF).
-  In the most common form of EA, the upper esophagus ends in a blind pouch and the TEF is
connected to the distal esophagus (type C).
-  Fifty percent of infants are nonsyndromic without other anomalies, and the rest have
associated anomalies, most often associated with the VATER or VACTERL (vertebral, anorectal,
[cardiac], tracheal, esophageal, renal, radial, [limb]) syndrome.
- Maternal history of polyhydramnios is often present.
-  The neonate with EA typically has frothing and bubbling at the mouth and nose after birth as well as episodes of coughing, cyanosis,
and respiratory distress, sometimes with abdominal
distension. Feeding exacerbates these symptoms, causes regurgitation, and can precipitate
aspiration.
-  In the setting of early-onset respiratory distress, the inability to pass a nasogastric or orogastric
tube in the newborn suggests EA
-  Plain radiography in the evaluation of respiratory distress might reveal a coiled feeding tube in
the esophageal pouch and/or an air-distended stomach, indicating the presence of a coexisting
TEF
-  Initially, maintaining a patent airway, pre-operative proximal pouch decompression to prevent
aspiration of secretions and use of antibiotics to prevent consequent pneumonia are
paramount. Prone positioning minimizes movement of gastric secretions into a distal fistula,
and esophageal suctioning minimizes aspiration from a blind pouch
- Surgical ligation of the TEF and primary end-to-end anastomosis of the esophagus via rightsided
thoracotomy constitute the current standard surgical approach.

Correct Answer. a,c,d,e

(236). True statement(s) regarding hypospadias?

a. Urethra opens on ventral aspect

b. Chordee always seen on dorsal aspect

c. Associated with deficient corpus spongiosum

d. Proximal variety is more common

e. Associated with penile torsion

Copyright © 2014 Delhi Academy of Medical Sciences, All Rights Reserved. 103/110
Solution. AE
Explanation: Hypospadias
- Hypospadias is a urethral opening on the ventral surface of the penile shaft affecting 1 in 250
male newborns. Typically an isolated defect, but its incidence is increased in disorders of sex
differentiation, anorectal malformation, and congenital heart disease.
- Usually there is incomplete development of the prepuce, called a dorsal hood, in which the
foreskin is on the sides and dorsal aspect of the penile shaft and deficient or absent ventrally.
- Some boys with hypospadias, particularly those with proximal hypospadias, have chordee, in
which there is ventral penile curvature during erection.
- Approximately 65% of cases are distal, 25% are subcoronal or midpenile, and 10% are proximal.
- Approximately 10% of boys with hypospadias have an undescended testis; inguinal hernias also
- Circumcision should be avoided, because the foreskin often is used in the repair in most cases.
- The ideal age for repair in a healthy infant is 6-12 months.
- Dennis-Brown operation is the procedure of choice
are common. Penile torsion, a congenital condition characterised by penis rotated to left
(counter-clockwise) is seen associated with many cases of hypospadias.
 Circumcision should be avoided, because the foreskin often is used in the repair in most cases.
 The ideal age for repair in a healthy infant is 6-12 months.
 Dennis-Brown operation is the procedure of choice

Correct Answer. a,e

(237). A healthy 64-year-old woman had a cancer of the left colon resected 4 years previously. During follow-up, an increased carcinoembryonic
antigen (CEA) level lead to a CT scan of the abdomen, which revealed two discrete lesions in the left lateral lobe of the liver. Liver biopsy
confirms that this is metastatic colon cancer. What is the most appropriate plan?

a. Inform the patient that there is no treatment, and that her expectation of life is limited.

b. Irradiation is recommended.

c. Local cauterization of the cancer is recommended

d. Liver resection is recommended

e. Chemotherapy is recommended.

Solution. D
Explanation: Before performing the left hepatic lobectomy, any extrahepatic metastasis should be
ruled out. If lung, bone, adrenal, or skin metastasis were present, then subjecting the patient to a
major operation would not be warranted in most cases. Moreover, before proceeding with surgery,
it must be ascertained that control of the primary tumor has been achieved and that the patient’s
physical condition will allow such a major operation. Surgical excision of hepatic metastasis results in
25%, 5-year survival. Patients not treated by hepatic resection do not usually survive into the first
year after clinical detection. Chemotherapy would be offered if resection were not indicated.

Correct Answer. d

(238). Indication(s) of surgery for polyps in Peutz-Jeghers syndrome include?

a. All patients even if asymptomatic need polypectomy

b. Obstruction

c. Bleeding

d. Adenomatous polyps

e. None of the above

Solution. BCD
Explanation: Peutz-Jeghers syndrome is characterized by polyposis of the small intestine and, to a
lesser extent, polyposis of the colon and rectum. Characteristic melanin spots are often noted on the
buccal mucosa and lips of these patients. The polyps of Peutz-Jeghers syndrome are generally
considered to be hamartomas and are not thought to be at significant risk for malignant
degeneration. However, carcinoma may occasionally develop. Because the entire length of the
gastrointestinal tract may be affected, surgery is reserved for symptoms such as obstruction or
bleeding or for patients in whom polyps develop adenomatous features. Screening consists of a
baseline colonoscopy and upper endoscopy at age 20 years, followed by annual flexible
sigmoidoscopy thereafter.

Correct Answer. b,c,d

Copyright © 2014 Delhi Academy of Medical Sciences, All Rights Reserved. 104/110
(239). True statement regarding medullary carcinoma thyroid is all except?

a. Associated with MEN 2B syndrome

b. Familial forms do not occur

c. Associated with MEN 2A syndrome

d. Seen in all MEN syndromes

e. Neural crest derived

Solution. BD
Explanation: Medullary thyroid cancers (MTC)
• MTC can be sporadic or familial and accounts for about 5% of thyroid cancers. There are
three familial forms of MTC: MEN 2A, MEN 2B,and familial MTC without other features of
MEN. Derived from parafollicular C cells of neural crest origin.
• In general, MTC is more aggressive in MEN 2B than in MEN 2A, and familial MTC is more aggressive than sporadic MTC.
• Elevated serum calcitonin provides a marker of residual or recurrent disease. All patients with MTC should be tested for RET mutations.
• In general, MTC has a poor prognosis as compared to papillary and follicular sub-types.
• The management of MTC is primarily surgical. Unlike tumors derived from thyroid follicular cells, these tumors do not take up
radioiodine.
• External radiation treatment and chemotherapy may provide palliation in patients with advanced disease.

Correct Answer. b,d

(240). Which among the following constitute grade V renal injury?

a. Main renal artery injury with contained haemorrhage

b. Parenchymal laceration

c. Completely shattered kidney

d. Avulsion of renal hilum

e. Laceration size > 1 cm depth

Solution. CD
Explanation:

Correct Answer. c,d

(241). Use(s) of accelerated partial breast irradiation includes?

Copyright © 2014 Delhi Academy of Medical Sciences, All Rights Reserved. 105/110
a. T1 breast cancers

b. Unifocal tumors

c. Multifocal tumours

d. Useful in stage 4 cancers

e. Useful in ER, PR positive cancers

Solution. ABE
Explanation: Accelerated partial breast irradiation (APBI)
• Accelerated partial breast irradiation is a type of radiation technique where a catheter is
inserted into the lumpectomy cavity and radiation is given to the tumor bed and adjacent
tissue rather than the entire breast.
• Indications: Only meant for T1, unifocal tumors which are ER, PR positive and have no LVI
(lympho vascular invasion)
• Advantage:
- Entire radiation can be completed in 5 days as opposed to 25 days for whole breast
irradiation
- Less total dose of irradiation
• Disadvantages
- More wound complications
- Slightly higher local recurrence rates if proper patient selection is not done

Correct Answer. a,b,e

(242). Differential diagnosis of childhood abdomino-pelvic tumours include all except:

a. Wilms tumour

b. Neuroblastoma

c. NHL

d. Hepatoma

e. CLL

Solution. E
Explanation:

Correct Answer. e

(243). All of these findings are consistent with a chronic unilateral urinary tract obstruction except

a. Anemia

b. Dysuria

c. Hypertension

d. Pain with micturition

Copyright © 2014 Delhi Academy of Medical Sciences, All Rights Reserved. 106/110
e. Pyuria

Solution. D
Erythrocytosis can develop in an obstructive uropathy as a result of increased erythropoietin production. Anemia in kidney disease occurs
as a result of progressive renal parenchymal destruction. As the kidney attempts to preserve renal function and expand blood volume,
renin levels increase and can cause a secondary hypertension. Dysuria can be seen in cases of chronic urinary tract obstruction due to
urinary stasis and the propensity to develop urolithiasis. Pain with micturition is a hallmark of vesicoureteral reflux, which causes a
chronic functional obstructive uropathy. Pyuria is common, as is urinary tract infection. Stasis promotes the growth of bacteria and
urinary tract infection.

Correct Answer. d

(244). What is not true of Spigelian hernia?

a. It occurs exclusively in males.

b. It involves part of the circumference of the bowel wall.

c. It is best repaired by the classical Bassini technique of inguinal ligament repair.

d. It occurs at the lateral edge of the linea semilunaris

e. It always contains the vermiform appendix

Solution. ABCE
Explanation: Spigelian hernia occurs at the semilunar line, which extends along the lateral border of
each rectus abdominis muscle. The posterior rectus sheath is deficient at the level of the arcuate line
(semicircular line) about one-third of the distance between the umbilicus and the pubic symphysis;
this is the most common site for Spigelian hernia to occur through the linea semilunaris. It occurs in
both sexes. The Bassini technique is applicable to inguinal hernias only. A hernia that involves part of
the bowel wall is known as a Richter’s hernia. The appendix may or may not form part of the
contents of the sac.

Correct Answer. a,b,c,e

(245). A 79-year-old man is examined for severe pain in the iliac crest. Metastatic disease from prostatic cancer is confirmed. What is the
treatment offered initially to most patients with
metastatic prostatic cancer?

a. Cortisone and pituitary ablation

b. Radical prostatectomy

c. Luteinizing releasing hormone (LRH) agonist (Leuprolide)

d. Local irradiation and testosterone

e. Hyperthermia

Solution. C
Explanation: Previously, androgen ablation was achieved by bilateral orchiectomy. However, total
androgen ablation is accomplished by oral administration of antiandrogens for 2 weeks followed by
injection of LRH agonist.

Correct Answer. c

(246). Risk factors for gastric adenocarcinoma include all except:

a. Atrophic gastritis

b. Smoking

c. Intestinal metaplasia

d. Hyperplastic polyps

e. More than 2 cups of tea consumed per day

Copyright © 2014 Delhi Academy of Medical Sciences, All Rights Reserved. 107/110
Solution. DE
Explanation:

Correct Answer. d,e

(247). True about surgical site infection (SSI)?

a. Can be due to environmental flora in OT

b. Can occur only upto 14 days of surgery

c. In case of implants, can occur upto 1 year of surgery

d. Staph aureus is common cause

e. Clean shaving can prevent SSIs

Copyright © 2014 Delhi Academy of Medical Sciences, All Rights Reserved. 108/110
Solution. AC
Explanation: Surgical site infections (SSIs)
 The CDC defines surgical site infections
incision within 30 postoperative days of the surgical procedure, or within 1 year if an implant is
left in place”
MC organism: Staphylococcus aureus (source is contamination from environment or OT
Types of SSIs: Superficial incisional, Deep incisional and Organ/space SSI OT
(SSIs) as “an infection that occurs at or near the surgical OT)

- Pre-operative hair removal and shaving has higher risk of SSIs. Hair clipping can be done but not shaving if hair removal is necessary.


Correct Answer. a,c

(248). A 40-year-old chronic smoker presents with ulceration of the tip of the right second, third, and fourth toes. He gives a history of recurrent
migratory superficial phlebitis of the feet occurring a few years ago. Physical examination findings are remarkable for absent bilateral
posterior tibial and dorsalis pedis pulses with palpable popliteal pulses. What is the single most important step in management?

a. Multiple toe amputations

b. Long-term anticoagulant therapy

c. Immediate operative intervention

d. Angiography followed by bypass surgery

e. Cessation of smoking

Solution. E
Explanation: This patient suffers from thromboangiitis obliterans (Buerger’s disease), a disease
found most frequently in white men between 20 and 40 years of age. It is a form of pan-vasculitis
involving the artery, vein, and nerve. Heavy tobacco smoking is strongly associated with this disease.
Early in the course of the disease, there is involvement of the superficial veins, producing recurrent
migratory superficial phlebitis. The distribution of arterial involvement is usually segmental,
involving the peripheral arteries. In the lower extremities, the disease occurs generally beyond the
popliteal arteries and distal to the forearm in the upper extremities. As long as ulceration or
gangrene is confined to a digit, amputation should be postponed as long as possible unless rest pain
or infection cannot be otherwise controlled. Bypass surgery is rarely indicated, and long-term
anticoagulation has not been of much benefit. The most important aspect of treatment is cessation
of smoking.

Correct Answer. e

(249). An 18-year-old man presents to the emergency department with a gunshot wound to the left chest in the anterior axillary line in the
seventh intercostal space. A rushing sound is audible during inspiration. Immediate management is which of the following?

a. Exploratory laparotomy

b. Exploratory thoracotomy

c. Pleurocentesis

d. Closure of the hole with sterile dressing

Copyright © 2014 Delhi Academy of Medical Sciences, All Rights Reserved. 109/110
e. Insertion of chest tube

Solution. D
Explanation: The immediate treatment is the closure of the hole by any means available. Sucking
chest wounds allow shift of the mediastinum to the opposite side. Thoracotomy is not usually
required. Laparotomy is indicated for a gunshot wound below the fourth intercostal space, but it
should follow respiratory stabilization. A chest tube will be required, following closure of the sucking
wound, to prevent a tension pneumothorax

Correct Answer. d

(250). Which of the following tumor markers is/are the best to monitor recurrence of colon cancer?

a. Alpha-fetoprotein

b. Carcinoembryonic antigen

c. Cancer antigen 15-3

d. 5-hydroxyindoeacetic acid

e. Calcitonin

Solution. B
Explanation: Carcinoembryonic antigen is not specific for colon cancer, but rising values after treatment are suggestive of recurrence.
Calcitonin is a marker for medullary Ca thyroid. Cancer antigen 15-3 is being studied in patients with breast cancer. Carcinoid tumors are
associated with elevations in 5-hydroxyindoleacetic acid.

Correct Answer. b
Test Answer
1.(b,e) 2.(a,c,d,e) 3.(a,b,d) 4.(b,c,d,e) 5.(b,d) 6.(a,d,e) 7.(c,d) 8.(a,c,d) 9.(a,b,c,d) 10.(a,c,d)

11.(d) 12.(a,b,d) 13.(a,d) 14.(b,c) 15.(b,c,e) 16.(b,d) 17.(b,e) 18.(a,c,d,e) 19.(c) 20.(a,b,e)

21.(b,c,e) 22.(a,c,e) 23.(a,c) 24.(a,b,c) 25.(c,d) 26.(a,c,d,e) 27.(a,b,d) 28.(a,b,d,e) 29.(a,b,c,d,e) 30.(a,c)

31.(a) 32.(a,b) 33.(a,d) 34.(a,b,e) 35.(a,b,d,e) 36.(b,e) 37.(a,b,d) 38.(a,b,d) 39.(a,c) 40.(b,c)

41.(b,e) 42.(a,d,e) 43.(a,b,c,e) 44.(a,c,d) 45.(a,b,c,d) 46.(a,b,d,e) 47.(b,c) 48.(a,b,c) 49.(a,b,d) 50.(a,d)

51.(a,b,c,d,e) 52.(a,b,c,d,e) 53.(a,e) 54.(a,c,d,e) 55.(a,d) 56.(a,b,c) 57.(b,d,e) 58.(a,c,d) 59.(b,e) 60.(b,c,d,e)

61.(d) 62.(b) 63.(a,c,d) 64.(a,b,c) 65.(a,b,c,d) 66.(a,c,e) 67.(a,b,e) 68.(a,b) 69.(a,b,d) 70.(a,b,c,d,e)

71.(a,c,e) 72.(c) 73.(b,c) 74.(a,e) 75.(c) 76.(b,e) 77.(a,b,c,e) 78.(a,b,d) 79.(c) 80.(a,b,c)

81.(d) 82.(c) 83.(c,d) 84.(c) 85.(a,b,c,d,e) 86.(a,b,e) 87.(a,c,d) 88.(d) 89.(a,b) 90.(a,c,d)

91.(a,b,c,d,e) 92.(a,b,c,d) 93.(a,c,d) 94.(b,c,d,e) 95.(d,e) 96.(c) 97.(b,c,d,e) 98.(c,d,e) 99.(d) 100.(a,b,c)

101.(b,e) 102.(b,c,d) 103.(a,b,c) 104.(a,b,d) 105.(a,b,c,d,e) 106.(b,e) 107.(c,e) 108.(a) 109.(b,e) 110.(a,b)

111.(a,c,d) 112.(a,e) 113.(b,c,d) 114.(b,c,e) 115.(a,b) 116.(d,e) 117.(d) 118.(b,d,e) 119.(a) 120.(a,b,c,e)

121.(c,d,e) 122.(a,d) 123.(c) 124.(a,b) 125.(d,e) 126.(a,b,c,d,e) 127.(c,e) 128.(a,e) 129.(a) 130.(b,c)

131.(a,b,c,d) 132.(a,b,c) 133.(d,e) 134.(a,b,c,d,e) 135.(a,c,e) 136.(a,b) 137.(a,b,e) 138.(d,e) 139.(b,c,d,e) 140.(a,b,c,e)

141.(a,b,c,d,e) 142.(a,b,e) 143.(c,d,e) 144.(d,e) 145.(a,b,c,d) 146.(c,d,e) 147.(c,e) 148.(b,e) 149.(a,b,c) 150.(a,b,e)

151.(a,d) 152.(a,c,e) 153.(c,e) 154.(c) 155.(a,c,e) 156.(b,c,e) 157.(a,c) 158.(a,b) 159.(a,e) 160.(d)

161.(a,b,e) 162.(a,b,c,d) 163.(a,c,d) 164.(a,c,d) 165.(a,c,d) 166.(b,d,e) 167.(b,c,d) 168.(a,b,c) 169.(a,c,e) 170.(a,b,c)

171.(a,e) 172.(a) 173.(b) 174.(a,b,d,e) 175.(a,d) 176.(b,c,d) 177.(d,e) 178.(a,b,c,d) 179.(a) 180.(a,b,c,d,e)

181.(a) 182.(a,b,c,d,e) 183.(a,c,e) 184.(b) 185.(c) 186.(a,b,e) 187.(a) 188.(a,b,d) 189.(a,b,c,e) 190.(a,c)

191.(a,b,c,d) 192.(a,b,d,e) 193.(a,b) 194.(a,b,e) 195.(e) 196.(a) 197.(a,c) 198.(b,c,e) 199.(b,c,e) 200.(a,d,e)

201.(a,b,c,e) 202.(b,c,d,e) 203.(b,c,e) 204.(b,c,d,e) 205.(b,c,d,e) 206.(a,c,e) 207.(d,e) 208.(b,c,d) 209.(a,b,d,e) 210.(a,b,c,d)

211.(b,c,d) 212.(c,e) 213.(b,c) 214.(a,b,c,e) 215.(a,b,c) 216.(a,d,e) 217.(d) 218.(c) 219.(a,c,e) 220.(a,c,d)

221.(a,b,e) 222.(b) 223.(a,d,e) 224.(a) 225.(b,c) 226.(b,d,e) 227.(a,b,c) 228.(a) 229.(a,c,e) 230.(b,c,d,e)

231.(b,c) 232.(c) 233.(a,b,d,e) 234.(b) 235.(a,c,d,e) 236.(a,e) 237.(d) 238.(b,c,d) 239.(b,d) 240.(c,d)

241.(a,b,e) 242.(e) 243.(d) 244.(a,b,c,e) 245.(c) 246.(d,e) 247.(a,c) 248.(e) 249.(d) 250.(b)

Copyright © 2014 Delhi Academy of Medical Sciences, All Rights Reserved. 110/110

You might also like